You are on page 1of 43

PART 2

ESTATE TAXATION
(Multiple Choice Questions)

A. COMPUTATION OF GROSS ESTATE

Benny Tai died leaving his daughter, Fina Tai, as sole heir to his residential
house and iot as his oniy property
Which of the following is noi an
element of the succession on his property,
f the adninistratrix of his estate is Mama Tai?
A. Benny Tai
C. Fina Tai
B. House and lot
D Mara Tai

2 inhentance doe not include -


A. property.
B. public office
. nghts not extinguished by death
D obligations not extinguished by death.

B
The right hold
to public office has no inheritability: it is not transmissible
mortis causa.
CPA IEWER IN TAXATiON
REVI
- Ampongan
ESTATE TAXATION- Multiple Choice Questions
64 65
Estate tax Is estate tax will be computed based on the
transmite 5, 2017.
prevailing law on September
it is imposed on the property
A a property tax because Tiitted by the Under the Benefit-Received Theory in estate taxation, the state
decedent to his hers D is a
Ban indirect tax because the burden of
paying the tax is shifted on the partner of the decedent in the distribution of the latters estate
of the decedent
executor or any of the heirs
c a n exase tax because it is imposed on the prvilege exercised to transs

over the estate A


ownership
a poll tax because it is also iniposed on residents of ihe Philicnin.
whether FHipino atzens or not pines n essentially
nortis causa transfer shall be
subject to estate tax
regardless of the title used in the deed.
Although there are secondary purposes of levying the estate tax, still the
main purpose for its imposition is to raise revenue.

7. in 2005, J Cruz gave a toan of P150,000 to


Sexy, his secretary in 2018, as
4 Which of the foliowing is not a distinction between estate tax and an act of generosity, J Cuz condoned the debt of
donor's t a Sexy in his last wili and
AThe tax imposed Is an excise tax testament. J. Cruz died in 2021. The condonation of the
debt of Sexy iS
(RPCPA)
8 Extens:on for payment.
CEfectivity of the iransfer of property. . A dona:1on inter VIvos
subject to donor s tax.
Penod for the fiing of return A payment or
compensaton for the services rendered
. A deduction from the
gross estate ofJ. Cruz
D. A donat:on mortis causa
subjeci
to estate tax.
A

D
Statement 1 The estate
tax accrues at the moment of
Statement 2. In estate taxation. the death of the decedent
taxpayer is the decedent 8 The foliowing are the motives of a
Which of the above
statements is correct? contempiation of death, except one
taxpayer that preclude the transfer in
Statement 1 only (RPCPA)
To
B
Statement 2 onty C.Both statements
D Neither statements To
relieve the taxpayer of the burden of
save income and management
To
property taxes.
avoid payment of estate tax
To make dependents
A finanCialiy independent

6Which of the
followng statements is C
An essentially mortis causa wrong?
A
3 In
shati be transfer which
subject to donor s ta has been titled as vivos defauit of
testamentary heirs, the iaw detemines who are to succeed to the
Estate tax is more of a inheritance of the
deceased. Which of the foliowing ranks first in the
f the revenue tax rather than succession? (RPCPA) order of
decedent died a
speciat ansfet of
pOssession of the September
property to
5. 2017
17 but the actual trar019, A legitimate children C. legitimate parents
the heirs took the B. surviving spouse
piace on
Jut 5 D illegitimate children
CPAREVIEWER IN TAXATION-AAmpongan
ESTATETAXATION-Multiple ChoiceQuestions 67
A The winnings in lotto and the interest on bank deposit are presumed 10
be cotnmun property ot the spouses.
children A and B H dted
are marned They have legitimateshouid be divided as fot ve
10 Hand W His estate of
P12.000 O00
ows
by W A and B 12 For estate tax purposes, the estate of the
decedent sha!l be valued at the time
Freeportion A of the preparation of the estate tax
return
P4.5 M P45M None B the e state tax is paid
3M
M M of death of the decedent.
3M 3M
6 M None Dthe estate is distnbuted to the heirs
Nons 6M
4.5 M 45 M 2.25 M
2 25 M
C

B
13 Mamo died leaving the
followng propertes
Tf there are fwo or more legitimate children or descendants, the surviving
to the legtime of each of the Stocks of Cruz Coporation (2.000 shares. Listed in
spouse stall be entified to a portion equal P 40. lowest P 39)
-
the Phisex (highest-
legitimate children or descendants. Common Stocks o' Hemo Corporaiion
stock exchange Cost P50 per
{1,500 shares) ot isted n ihe
snare Book value P45
-

11 Based on the foilowng deta. how much is the value ofthe decedent's interest 000. bcok value - F350 000 market value per
snare
-

Ca (cost - P800 P400


if he died March 31. 2018? Real properties (zonal va'tue P120 00
-

000
Cash in dank, joint account of the decedent and his wife P254,000
assessor s vaiue - Pr2,000)
The gross estate of Mamo is
intarest on the bank depost (Jan 1 - June 30, 2018) 9000
P619,500 P 624 00
Dividends from a domestic corporation. 60,000
Date of declaration - February 5 2018 867 500 666 500
Date of record Apni 15, 2018
Date of payment May 15, 2018
Share in 2017 net orofit of partnership distributed to D
partners on April 155 9,000 Shares Cniz Corporation i(40-39)/2x
Vinnings in iono Bet Marcn 30, Apri 3, 2018 araw) 500,000 Shares Hemo Corporation (P43 X
2,000 P 79.000

A P 383 750 Car 1500o 67.500


B. 138.000
CP145000 Real properties 400.000O
388,250 Gross estate
120,000
666,500
If the stocks are traded in the stock
exchange, the market value is thc
average of the highest and the lowest value of such shares at a datee
nearest the date of death, if none is available on the date of
Cash in bank death itself.

Interest
(254,00o/2)
(9,000/2 x3/6)
P 127,000O if the common stocks are not traded, the market value is the
of the shares on such date while unisted preierred shares are valued at
book value
Share in 2,250
partnership profit (9.000/2)
Lotto winnings 4.500
par value.
(500,000 / 2)
Decedent' s interest 250,000
in tiie casë ui reai
property, tBe iiarket value is the higher ainount
between the value as determiined by the Coinrissioiner of Internal
383.750
CPAREVIEWER IN TAXATION -Ampongan
ESTATETAXATION -MultipleChoice Questions 69
68 City Assessor
Revenue (zonal vaBue)
and the
Provincial or
(assessor's When the decedent is a nonresident alien (with reciprocity) only real and
value). tangible personal properties situated in the Philippines are subject to
estate tax. All other properties are exempt.
n the Phiippines. The propertiee

died
Ukrain1an,
the Philippines iated
14 Utyanov Kerivsky. not be subject
to estate tax in
country will was 17 If the decedent was a nonresident allen (no reciprocity), how much is the gross
in nis own
estate?
C. Nonresident citizen
Resident citizen P3,725,0c0 P 500 O00
D. Nonresident alien
Resident alien 975,000 475,000

D
D
Whenever a decedent is a nonresident alien (no reciprocity). al! propetties
the folowing information: property and
items 15 through 17 are based on
situated in the Philippines (realarepropertv. tangible personai
incdudible in his estate.
in the United States with the following properties intangible personal property) gross
Dina Mathay. filip1na. died
Condominium unit in New York City P 2.000.000 18. Which of ihe following is en intengibie personal property within?
Shares of stock in a foreign corporation 600,000
475,000 A. Franchise exercised in the Urnited States
Interest in a partnership, domestic
Bank deposit in a New York City bank 150 000 e Shares or nghts in a domestc business partnership
500 000 Bonds isSed by 2n american corporation.
Carin Cebu denated inter vives 5 years ago to her son
Stocks issued by fore:gn corporat:on with business situs in the Phitppines

15 Which property shoutd be inctuded in her gross estate7 3 only C All of the above croperties
B and D D None of the above properties
Al of the above properttes
Only the properties ocated in the Philippines
CAll the above properties except the car
The properties igcated in the Philiopines evcent the intangihes

19 An example of intangible personai property without is

C A Domestic shares of stock


BForeign shares, 85% of the business of the corporation is in the Phiippines
Fore gn shares with business situs in the Philippines
hross
e dcedent was a nonresident aien with reciprocity). is
how mun the are kept in Makat:
estate? Foreign shares, cert:ficate ofstock

AP3.725 000
975 000
P 500,000
None D

D
CPA REVIEWER IN TAXATION- 71
70
donations is not
inciuded as part ofarose.
Ampongan ESTATE TAXATHON
Mulliple Choice Questions
about to,die
of cancer,
20. Cne of the following estate inTofmed by his physician
that he was

Albino was
When
Arevocabletransfers 23.
i s properties: Market value-
reservation of certain rights ne sold
Selling
transfers with Market value
B. Date of death
transfers under special power of appointment Date ofsale Price
C. contemplation of death P2,700,000
transfers in P2,500,00O P1,500,000
D. 300,000
Land 500.000 300,000
Jeweiries 220.000 250,000
Shares of stocks
200,000
C limited power 800,000
Transfer under 600,000
1,000,00O
In a special power of appointment, the decedent is considered onlv an of appointment
incliuded in the gross
trustee to the property. Hence, said property should not be a partofhis much should be
data given, how
From among the
death?
estate. estate of
Albino upon his
C. Pi,430,000
A. P1.200,000 1,400.000
21 Which of the following transfers is included in the gross estate? 1,230,0C0

A. transfer inter vivos


8transfer under general power of appointment
. transíer under special power of appointment A
cteatih Seliing price GrOSS estate
D. transfer for an adequate and full consideration Mupon
P2.700.000 P,300,000 P1,200.00o

Land 300,000 30,0


B
Jewciries 250.000
220,000
hares ot stoCks
300,000 b00,000
Inageneral power of appointment, the decedent is practiclly the owner Liniited power of apnointment ,200,00
of the property, and
so the Tota
property is deemed part of his gross estate.
sold to
was about to die of
a iiver cancer, Bongbong
22 Decedent Jose Llamado. had the
following data 24 On the bel.ef that he
valued at P1,100,000 for
the same amount. Six months

Value of the property at the time of Eengheng a property that t1me, the property
had aBready
car accident. At
sale P 1,200,000 iaier, Bongbong died of a tax purposes, the
amount
Vaiue of consideration when sold For Philippine estate
1,000,000 a value of P1,300,000.
Value of property at the time of of Bongbong is
dez includibie in the gross estate
1,500,000
The amount includible in C. P 200,000
the gross estate is -

A. P1,100,000
A. P 300,000 D. None
C. P 200,000 B 1,300,000
B 500,000 D. 1,500,000

B D
estate becatuse the sale of the
No amount shall be incduded in the gross consideration.
Value the time of
at
Less: Considerationdeath made for a tull and adequate
was
P 1,500,000 property
Amount to be included in favor of
his
gross estate 1,000.00 25 February , 2009, Angei
prepared a wii orn his property
children Bersabe and
500,000 died September 5,
2009 survived by his
children. Ange!
TAXATION- Amn
ngan ESTATETAXATION-Multiple Choice Questions
73
REVIEWER IN
CPA
made an extraj silent as to whether the beneficiary is revocable or
72 When the policy is
poSsession
and made
the Philippines provides that it
the 1s
immediately
took o v e r the
witnout
registering irrevocable, the Insurance Code of
who
20, 20O09 Dutproperty to Contado on the gross estate _ubject to estate tax.
revocable; hence, inciudible in
Contado
September 2010
Which dat
on sold the
parttion Bersabe
of Deeds. hospitalization.
his Ould be
Register
finance the
expensesfor of computina insurance not payable to estate, executor or
administrator
i n o r d e r to the property Tor purposes estate 28. Proceeds of life
estate if the beneficiary appointed in the policy
in valuing in the gross
used as
the basis shall be excluded
tax ? September 20, 2009 is
February 1, 2009 iviay 7, 2019 A. Revocable C. Irrevocable
Saptember 5, 2009
Revocable or irrevocable
D. the executor
B

L B
be based on the fair market value at the
:
The vaiue of the property should property.
owner of the
death of the in the taxable gross astate (RPCPA)
time of
29 Proceeds of life insurance inciudible
his executor or administrator proceeds from SSS or GSIS.
the estate of the deceased, A. insurance
Amounts received by
26.
under policy taken by
the d e c e d e r n t upon his own life is
B. Amount receivable by any beneficiary irrevocably designated in the
as an insurance
policy by the insured
(RPCPA)-
C. Amount receivable by any beneficiary revocably designated in the
A. excuded from thegross estate. nsurance policy.
whether he beneficiary is revocable or its
B. part of the gross estate DProceeds of a
grou insurance taken out hy a company for
iTevocable.
is revocable. empioyees.
C.part of the gross estate if the beneficiary
irrevocable
D.part of the gross estate if the beneficiary is

30. Which of the foliowing proceeds of life insurance policies is exempt from
estate tax?
27. Case 1- Designation ofthe beneficiary is revocable
L i f e insurance policy on the life of Kristine, appointing her sister as the
Case 2-Designation ofthe beneficiary is irrevocable.
Case 3- Policy is silent as to whether the designation is revocace irrevocabie beneficiary.
irrevocable tLife insurance policy on the life of Kristine. appointing her brother as

the revocable beneficiary.


in which of the above cases will the proceeds be exempt from estate t a of Kristine. eppointing her executor as
ne
L i f e insurance policy orn the life
that insurance proceeds is neither
assuming the beneficiary of the life
estate, the executor nor
the irrevocabie beneficiary
the administrator of the estate? IV Life insurance poiicy on the life of Kristine, appointing her children a s
A. Case 1 beneficiaries. The policy is silent as to whether the appointment is
only C. Case 2 only
B. Cases 1 and 3 revocable or irevocable.
D. All of the above cases
C. II and !!
only
i and IV/ All of them
C
CPAREVIEWER AXATION-Ampongan
74 ESTATE TAXATION - Multiple Choice Questions 75
A
X devised in his will real property to his brother Y who is entrusted with
P500,000 with Philh the obligation to preserve and transmit the property to Z, a son of Y, when z
life for
31.
Bodo-bodol
insured his

Are the proceeds


subject to estate
tax Insurand becomes of age. he transmission from Y to his son Z is subject to tax.
Company. (RPCPA)
the event of death of Bodol.B
his estate in uo1? A. First statement is correct, second statement is wrong.
. if he designates as the irrevocable
benefician
his estate
if he designates B. Both statements are not correct.
i1.
his brother as the irrevOcabie beneficiary2
. if he designates C. Both statements are corect
his emp1oyer in nis tavor as the
the irrevOcable
was taken by D. First statement is Wrong. second statement ie correct.
V if the policy
beneficiary?

C. Yes, No, Yes, No


A. Yes, Yes, No, No A
D. No, No, Yes, Yes
B. Yes, No, No, Yes
The transmission from C to B is a merger of usufruct in the owner of the
naked title. It is not subject to estate tax.
A The transmission of the reat property fron Y to Z constituies
fideicommissary substitution which is exempt fron estate tax.
The proceeds are taxable it taken by the insured !n his own life as the
irrevocable benefciary. Ifit is taken by one in the life of another, it is not
34. One of tthe following is included in the gross estate
taxable, whether the designation of the beneficiary is revocable or not.
A. Benefits received from GSIS
B Benefits received from U.S. Veterans Administration
32. The foiowing are transactions and acquHsitions exempt rom transfer tax
C. Benefts received from damages during Worid War 2
except (RPCPA) Benefits received from a tax exempt employer as a consequence of
D
A Transmission from the first heir or donee in favor of another beneficiary death of the employee.
in accordance with the desire of the predecessor
8. Transmission or delivery of the inheritance or legacy by the fiduciary heir
or legatee to the fideicommissary. D
C. The merger of uaufruct in the owner of üie naked üle
D. All bequests, devisees, legacies or transfers to sociat welfare, cultural and 35. Wnich of the following distinguishes conjugal property from community
charitable institutions. property?
A Properties inherited during mamiage.
D B Those acquired ihrough occupaion during mariage
Fruits of property acquired thru donation during the marriage
Transfers to social welfare, cultural or charitable institutions D Income earned by each spouse during marriage
are ea
rom estate tax if it is indicated that not more than 30% of the donan
used for administration purposes.
C
33. A devised in his will a piece
of land; naked title to B and
usufruct hiect In conjugal partnership of gains, the fruits of exciusive properties are
long as C fves, thereafter to B. The in absolute community of property regime, the fruits of
to estate tax but the transmission from A to B and an the conjugal, while
merger of the usufruct and the naked titie to o exclusive properties are exclusive properties aBso.
death of C is exempt.
76 CPA REVIEWER IN KATION-Ampongan
Malakas is married to Maganda. From among the properties below,
ESTATETAXATION Mutiple Choice Questions 77
36
considered as their conjugal property? which D
one is
as his or her own
A That which is brought to the marnage
BThat which each acquires dunng
the marrage by inheritance 40. A. Share of the decedent in the community property.
C The fruits of an excusive property. B. Share of the surviving sp0use in the community property
D. That which is purchased with tne exciusive property of the Wife.
C. Exclusive property of the decedent.
D. Exciusive property of the Surviving spouse.
Which of the above properties are incuded in the grosS estate of the
decedent?

A. A and B C. A and C
37. One of the foliowing is not a community property of the spouses B.A, B and C D All of the above properties
A. Property inherited by the husband before marriage.
B. Winnings in gambling.
C. Fruits of property inherited during the marriage.
D Fruits of property inherited before the
marriage.
41 Properties acquired by gratuitous title before the mariage are generaly
ciassified as
C
A. Community properties under absolute community of property regime.
B Conjugal properties under conjugai partnership of gains.
38. Fat Tai died. From among the
properties enumerated below, which
considered as part of his gross estate? one is not Which of the above statements is correct?
A. conjugaB property A A oniyy C. B only
community property BA and B D Neither A nor B
exclusive property of the decedent
D. Xdusive property of the
surViving spoUse

D
Numbers 42 through 45 are based on the following irformation

39. When a
person dies and Aldo died teaving the following
between the husband and during the mariage the property relations onship properties
the gross estate of the wife was that of (a) Condo unit in Baguio City, brought into the mamiage P300,000
the decedent conjugal partnership or gau
would include (b) Rent income of condo unit in Baguio 60,000
His exclusive (RPCPA) (C)Apartment unit in Cebu City. brought into mariage by wife
8. His exclusive properties only. (d) Income of apartment unit in Cebu City
240.000
C. All the properties and one-haif of the 25,000
D. properties of husband and
His exclusive conjugal properties e) House in Camarines Sur, acquired by Aldo during mariage 375,000
wife
properties and all conjugal (f) income of house in Camarines Sur 50,000
properties. g) Land in itoilo City, earnad by wife during the mariage
(h) Income land in loilo City
225,000
80,000
T a x i in Manila, inherited by Aldo duning mamriage 500 000
G) income of taxi 175,000
CPA REVIEWER IN TAXATION=Ampongan ESTATETAXATION-Multiple Choice Questions 79
78 stocks in
Japanese
Corporation,
inherited by wife Bank deposit in Canada 350,000 1250.0000
)Shares
of 430.000 Gross e s t a t e 2,340,00o0
during mamage
from Japanese
Corporation
Aldo before
85,000
Drvidends inhented by of
bus in Dagupan
City, community of property regime, the total community property
(m) Passenger 100 000 44. Under absolute
marmage
10,000 the spouses IS
of bus in Dagupan City
C. P2,495,000
(n) income
in Canada,
donated to Aldo before marmiage 350,000 P1,820,000
D 1,170,000
Bank deposit 1,990 000
()
bank deposit in
Canada 5,000
interest on
p)
value of conjugal properties
of gains, the total
42 Under the conjugal partnership
of the spouses is
C P1.090,000 Condo unit in Baguio City P 300,000
AP1.170,000
2,495,000 Rent income of condo unit in Baguio 60,000
B 1,820000 Apartinenl umt in Cehu City 240,000o
neome of apartment unit in Cebu Ci 25,c00
House in Camarines Suur 375.000
income of house in Camarines Sur 50,000
Rent income of condo unit in Baguio P 60,00o Iand in I!oilo City, earned by wile during marnage 225.000
income of apartment unit in Cebu City 25.000 income of land in Iloilo City 80,000
the martiage 375.000 Passenger bus in Dagupan City 100,000
House in Cam. Sur, acquired durnng
house in Cam Sur
GO.000 Ineome of bus in Dagupan City 0,000
ncome of 225.000
Land in Iloilo City, earned by wife during marriage eposit in Canada, donated to Aldo betore marmage 350,000
Income of land in Iloilo City 8o.000 Inierest on bank deposit in Canada 5000
Income of taxi in Manila 175,000 Total community properties 1.820,000
85.000
Dividends from Japanese corporation
Income of bus in Dagupan City 10,000D 45. Under absoiute community of property regime, the amount of gross estate of
Interest on bank deposit in Canada 5.000 Aldo is:
Totai conjugal properties L,090,000
P1,990,000
A P1,170,000
2.495 000 1,820,000
0
43. Under conjugal partnership of gains, the gross estate of Aido amounts
A P2,340,000 C. P 1,990,000
B 2495,000 D. 1,995,000 B
Community properties P1,820,00
Add: Exclusive properties of Aldo
A
Inherited during marriage (Taxi) P500,000
Income of taxi in ManiBa 175.000 675.00
Conjugal properties P1,090,000 Gross etate 2.495000
Add:
Fxclusive properties of Aldo
Condo unit in
Taxd in ManilaBaguio P300,00o
Bus in Dagupan 500,000O
100,000o
CPA
REVIEWER INTAXATION-AmDon
80
Pilar on January 20, 2018
without
pre-nuptial ESTATE TAXATION- Multiple ChoiceQuestions 81
Pepe married ement
an old Spanish house in Vigan, ilo
Pepe brought into the mamage
P2,000,000 while Pilar brought with her a 200-hectare pineapplenl worthin B
when she was still single.
Bukidnon which she acquired
49. The deposit with Metrobank
git rom ner parents another
She received donation propter nuptias as 200
hectare banana plantation
in Davao City on January 31, 2018.
died of car accident. The joint account D
Three (3) years later, Pilar a
was P5,000,000.
deposit
of the spouses with Metrobank
She was insured with an insurance company for P2,500,000 with Po 50. The proceeds of the insurance policy is -
the appointed irevocable beneficiary
Pepe a
Excluded from gross estate
3. included in the gross estate
50, the properties identiied above by choosina
For numbers 46 to classify
below:
Deductibie from gross esiate
your answer from the options None of the above

Options:
a. Exclusive property of Pepe A
D.Exclusive property of Pilar
C.Conjugal property of Pepe and Pilar
d. Community property of Pepe and Pilar tems 51 trirough 54 are based on the tollowing data

Angelo married to Ange! 3 years


ago. died ieaving the following propertes
46. The old Spanish house in ilocos Sur
Condo Unit at West Tower Condominium Makati Cty.
acquired by him and his wife P2 500 000
Apartment unit in Vancouver, Canada inherited from his
D parents wha died 2 % years ago 3 500,000
Voivo car registered in Canada, donated to him
Pepe and Pilar are under the absolute con1munity of property regime by his
motiher four (4) years ago
hecause the marriage was celetbrated after August 3. 1988.
Toyota Fortuner in the Philippmes, purchased by Angelo
200000
Allmarriages celebrated on or after August 3. 1988 shall be govemedby out of his Sxc!lusive propeity
,200,000
the absolute community of Jaweiry in the Philippines, inherited last year by his wife
property regime, there being no pror
agreement in writing as to their property relations during the marnage Angel, from her mother 550 000
Cash in bank - Banco de Oro, 50% was eamed by Angeio
47. The banana before marriage, 50% was earned by the spouses 840,000
plantation in Davao City Interest on bank deposit (net of with hotding tax) 8 000
nterest in a domestic partnership acquired by Angei before
B marriage 300,000
investmernt with Acer Corp, foreign corporation 85 5% of
business is in the Philippines 1,000 0000
48. The income of the Dividends with Acer Corporaton, date of record was made
banana plantation after death of Angelo 45,000
CPA REVIEWER IN TAXATION-Ampongan
82 ESTATE TAXATION- Multiple Choice Questions
investment with Filipinas Company,
exchange
domestic 25,000
(highest: P23.00:
83
in the stock
shares, traded A. P 10,127,625 P 3,700,000
lowest P22.83)
Filipinas Company,
date of record, one B. 11,827,625 D. 6,577,625
Drvidends from
of dividends tax)
month before Angeio's death (gross 7,500
for an accident
Recevable on a foreign insurance company
SiX months before de ath B
insurance
suffered
50,000
Proceeds of a life insurance taken by the employer
Corporation of Angelo on
his life
200,000 Condo unit, Makati
Conjugal Exclusive
Receivabie on ife insurance taken by Angelo
on his own 2.500000
Apartment unit, Canada
the iTevocable beneficiary 3,500,00o
life appointing his estate as Voiro, Canada
were used in paying the 2,000,000
common funds of the spouses Toyota Fortuner. Philippines 1,200,00
insurance premium 150,000 Casi with Banco de Oro (840,0o0o/2) 420,000 420,000
51 The amount of gross estate if Angelo was a non-resident citizen ungar tha
Interest On bank ieposit 8,000
ne Investment Acer
absolute community of property regime
-

Corporation 1,000,000
investment - Filipinas Company 572,875
Diviciend - Filipinas Company
P 12,127,625 P8.577,625 750
B 11,827,625 3,700,000 Recevable
Receivabie
irom
on
a toreign insuranee company
iife insurance, estate 18
50,000
benetiCiary 150.000
Totais 4.707.625 7,120,000
A Gross estate (4,707,625 +
7,120,000) 11,827.625
Communily Exclusive
53. The gross estate if Angelo was a non-resident a!lien without reciprocity under
Condo unit, Makati 2,500,0000 the absctute community of property
regime
Apartmet unit, Canada 3.500,000
Volvo, Canada 2,000.000 AP 10,127,625 . P 3,700,000
Toyota Fortuner, Phiiippines 1,200,000
11,827,625 D 6,577.625
Cash with Banco de Oro S40,00o
Interest on bank deposit 8,000
Interest in a domestic partnership 300.000
Investment -Acer Community Exclusive
Corporation
Investment Filipimas Company
1,000.000
Condo unit, Makati
-

P2,500,000
(23.00+ 22.83) 2] x25,000 572,875
Toyota Fortuner, Philippiries P1,200,o00
Cash with Banco de Oro
Dividend-Filipinas Company (7.500 x90%) 6.750 Interest on bank deposit
840,000
Receivable from a foreign insurance company 50,000 Interest in a domestic partnership
8,000
Receivable on life insurance, estate is 300,00o
Totals beneficiary 150.00 4.700,000
investment Acer Coporation 1,000,000
7,427,625 investment Filipinas Company
-

572,875
Gross estate (5427,625+4.700,00o) 12.127.625 Dividend Filipinas Company
-

6,7550
52 Receivable on life insurance, estate is beneficiary
Totals
150.000
The grOSs
of gains
estate if Angelo was a jugal partnership
resident alien under the conjugan
5,377,025 1,200.0po
Gross estate (5.377,625 + 1,200,000) 6.577625
84 CPA REVIEWER IN TAXATION-Ampor ESTATETAXATION Multiple Choice Questions 85
was a
non-resident alien, with reciprociity, under
54 The gross estate if Angelo 115,000 45,000
of gains 160,000 None
the conjugal partnership

A. P 10,127,625
P6,577,625
11,827,625
3,700,000.
C
Principal P 100,00o
D Interest (Oct -- Dec) {P1o0,000 x 5% x3)
15000
Conjugal Exclusive Exclusive properties 115,000
Condo unit, Makati 2,500,00o

Toyota Fortuner, Philippines 1,200,000 Conjugal (9 months) (100.000x5%x9) 45.000


Totals 2.500,000 1,200,00o Whenever an amount or eredit payable within a period of time belongs
o one of tlhe spouses, the sums which may be collected during the
3.700.000
Gross estate (2,500,000 +1,200,000) marriage in partial payments or by instailments the principal
on
shall be
ihe exclusive property of the spouse. Hewever. interests faliing due
a parcel of land on installment basis
55. Sol, Terito, a bachelor, bought during the marriage on the principai shal belong to the conjugal
deed of sale, the ownership shall be
measurig 200 square meters. In the partnership.
A year before
vested in him even before ful payment of the purchase price.
to Roma Raga and out of the
he could fully pay the price, Sol got married
the balance was paid. Who owns the land? 57 Ivan and Sarah are marnied van inherited a residential lot
P1,000,000 from his parents. Out of the conjugal funds the spouses
valued at
conjugal funds,
fund Constructed a house on the land had which cost them P2,000 000. Who
A. Sol because he paid more than what was assumed by the conjugal
owne the land immediately upon the death of either the wife or the husband?
B. Roma because she is the wife of Sol.
How about the residential house?
C. The conjugal partnership because the full payment was made duning
the marriage. Residentiai lot Residential house
D. Sol because the ownership was vested to him before the marriage. Conjugal
Conjugal
3 ExcluSive of lvan Conjugal
CExclusive of lvan Exclusive of lvan
EXCusive of lvan
D D.Conjuga
The determination of ownership depends upon the time it was vested Dy
ue
the conjugal fund. lf full owmership was vested before marriage, B
property is exclusive of the buyer. Otherwise, it is conjugal. When the cost of the improvement made by the conjugal partnership
and
at
inerease in value are more than the value of the property
any resulting
of the spouses shal
56 Rufino is engaged in the business of lending money at a usunoU5 the time of the improvement, the entire property
to reiinbursemeni of the value
interest. On September 30, 2018, he lent P10,000 to Pepito payabe to belong to the conjugal partnersiip, subject
at the time of the impro vement;
1) year at 5% interest per month. On December 30, 2018 hegotnaety ot the property of the owner-spouse
in ownership by the owner
Rica Gomez. Based on the above data, how much is the exclusive plO otherwise, said property shall be retained
reimbursement of the cost of the
to
of Rufino? How about the spouse, likewise subject
conjugal property? mprovement.
Exclusive Coniugal
A P 100,0000 P 60,000
B. 115,000 60,000
CPA REVIEWER IN TAXATION- Ampongan
ESTATE TAXATION Multiple Choiee QuestioIs
1he ownership
of entie
tihe property shall be veste l upon th
shall be nade at the time of the
87
burseent, which liquidation of the
nugai ptezship.

Tong Siok a Chinese bilionaire


and a Canadian resident dia. B. DEDUCTIONS FROM GROSS ESTATE
valued at P$0 Diion snd in the Fhikippines assat
assets in China
astate tax purpOSeS the allowable dedivkat
P20 Dilion For Phippina for
axnenses losses, indebtednoSS, N1d tRAES, Pparty previously
ransfers for public use, and the
siare Of i s
surviving spouse n 59 Which of the following tems is deductible the grosa?
coniugal partmership amounted to Pi5 Dilion. tong's gross estate
is
for A Funeral expenses Medical expensen
hihppine estate tax purposes
JudCal expen8es Standard deduction
P 20 bilion P 100 bilion
5 billion 85 bitlion

60 1hree of the four items of deduction below an requiod to be ichuded n the


A gross estat Which one is not?

A Standard deduction
B CAams aganst insolvent persons
Cfeneffs recervedunderRA 491/
T e untimnshed vialue of tthe pr0perty nort)agud

61 Requsites for deductibility of "claims ayainst the estate" exCept


A The liability represents a personal obligation of the deceased oxistung at
thee time cof is death
e debt may or mnay not have boen condonad.
he lability must thave bven cOrtracted by the deeasd m good fat
and for aNI Hdequale cONaderation m monoy or monay s worth
The clam must be a debt or claim wBuch i8 valid in law and entorcaaie

ia CONRICT about "Clama Against the


62 Which of the tollowing atatementa
Estate"?
CPA REVIEWER IN TAXATION Ampongan
88 is secured by a mortgage c ESTATE TAXATION=Multiple Choice Questions 89
the decedent which
contracted by
A A loan as deduction from gross state falling of
still be claimed A
his iot can
the estate. under
claims against
tax and real
estate taxes accrued betnrs
that the death,
B. Unpaid income lifetime are deductibletfrom the
65 All of the folloWing. except one, are deductible from the gross estate of a
gross estat of
being payable during
"claims against the estate".
the decedent who died September 30, 2019
decedent as
the estate, as
deduction from the aron.
A. Income tax on income earned from January to September 29, 2019.
Claims against
reprecents obligations enforceable during the lifetime of tha dstate
dent. B. Gift taxes on donations given June 12, 2019.
the lifetime of the decedent which is Real property iaxes payable during the iast quarter of 2019
D.Debts contracted during n
the decedent debtor died on the second vea D. Income tax on income earned during the last quarter of 2019.
three (3) years but not
estate.
deductible from the gross

C To be deductible, taxes must accrue before the death ofthe decedent. Real
time of death of tho property taxes accrue on the 1st day of January of every year although the
which is secured by an unpaid mortgage the
at
A ioan payment is allowed to be made on staggered basis in the succeeding8
an unpaid mortgage and not as claims against
decedent is deductible as quarters ot the current year.
the estate.
and real estate iaxes that have accrued prior to
Similarly, unpaid income 66. Which of the foilowing options is CORRECT? in ciaims against insolvent
deductible as unpaid taxes and not as claims
the death of the decedent are
persons
against the estate. as deducion from gross estate, only the entire amount of uncollectibie
A.

in order that claims against the decedent's estate


clams shail
be included in the gross estate of the decedent
as deduction from gross estate, the fu!i amount of receivable inciuding
63. The following requisites
are B.

may be deductible except (RPCPA) uncoliectible, must be included in the gross estate.
C. such ciaim is not deductible from the gross estate if the decedent was also
A. They must be existing against the estate. insolvent at ihe time of his death.
B. They must be reasonably certain as to amounts. Dii the entire debt is uncollectibie, it may be omitted in the gross estate
and
They must have been prescribed. as deduction therefrom.
They must be enforced by the claimants.

B
entire amount of cdaim, whether
n ciaims against insolvent persons, the
in the gross estate, regardless
collectible or uncollectible, must be included
decedent-creditor.
64 One of the following is deductible as "Claim against the estate? of the solvency or insoivency of the
died.
A. An obligation contracted by the decedent one (1) day beror Was
loan is merely an accommodation
B. An obligation of the decedent which prescribed while the deceu 67. Statement 1: f the proceeds of a mortgage receivable amount
as a
still alive. value must be included in the gross estate
oan, its
not reduced in writing under the staui and as a deduction thereof.
O0gation which was
Fraud.
D. An obligation which shall be paid by the heirs.
CPA REVIEWER IN TAXATION-Amr :STATETAXATION-Multiple Choice Questions 91
90 to recognize the
he a .

Statement 2:
if there is legal
r e c e i v a b l e of
impediment

the estate,
id
the unpaid mortgage payabl commodaton
shall not be
Taxes inust have acerued before the death of the decedent. Taxes on
income ot properties which have accrued after death are not deductible.
ioan as
deduction from
the gross estate.
aflowed a s a
is correct. 69 Wnich of the following siatements is INCORRECT regarding an unpaid
statement
A. Only the first corect.
Both statements are nortgag8?
B is correct.
second statement
Only the A. Unpaid mortgage, as deduction from the gross estate of a resident citizen,
Neither statement
is correct.
D. must pertain to a property mortgaged which is situated either within or
without the Philippines.
BUnpaid morigage, as de om the gross estate of a non-resident
alien, must pertain to a mortgaged property which is situated within the
Philippines only.
left C. If the unpaid mortgage was contracted by a prior decedent on a property
foliowing expenses and obligatons
were by Boning upon his death.
68 The
th inherited by the present decedent before marriage, it is chargeable against
Notes payable, not notarized P 30,0000 the community property f it benefited the community property of the
Loans payable, PN8 300,000 spouses
Accounis receivable, debtor not insclvent 40.000 f the unpaid mortgage was contracted by a prior decedent on a property
Accounts receivabBe, debtor is insolvent 60,000 inherited by the present decederit during marriage, it is chargeable against
Death benefits from employer 200,000 the community property of the spouses if it benefited the community
Mortgage paid 50,000 property of the prior decedent.
income taxes on income of decedent's estate 7,500
The amount deductibie from gross estate is -

A. P600,000 P 560,000 An unpaid mortgage on a property abroad of a non-resident alien decedent


8. 550,000 D 5,560,000 is not deductible because the property subject of mortgage is not includible
in the gross estate.
An unpaid mortgage maybe classified either as an exclusive or a
community property deduction depending upon which property was
Loans payable P 300,000 benefited by the proceeds of the mortgage. Thus, if it had the benefited
Bad debts family and/or the community property of the spouses, such unpaid
60,000
RA 4917
200,00
mortgage is deductible from the community property. Otherwise, it is
Standard deduction deductible from the exclusive property of the decedent.
Total deductions 5.000,000
5560,000
To be deducti ble, claims against the estate out of debt instrnument must
70. Casualty losses are deductible from the gross estate if (RPCPA
be duly notarized. 1 Statement: Such ioss was incurred during the settlement of the estate
2d Statement: Such loss was incurred not ster than the last day for the
Caims of the
debtor is
estate against other
persons are deductible Oly
the
deciared insoBvent. payment of the estate tax.
Mortgages paid are allowed only as deduction from the the A. Both statements are false.
property in computing a vanishing deduction. In vauhe net B 1s statement is false, 2nd statement is true.
estate, the deductible item is compus C. 1st statement is true, 2nd statement is false.
unpaid mortgage. DBoth statements are true.
92
CPA REVIEWER IN TAXATION-Amp
Ampongan ESTATETAXATION=Multiple Choice Quiestions 93
73 Which of the following losses is deductible?
D
A Destruction of a house by an earthquake which kiiled the decedent
January 5, 2020 and his estate inci which before the death of the decedent but was onty
filipino resident,
died on
losses B Shipwreck occurred
71. Y, a
discovered after his burial.
dueto of a car in an accident but was fuly compensated by a
1st loss: From fire on February 2, 2020 of improvements on his propert Totai wreckage
c o m p r e h e n s i v e insurances.
insurance.
not compensated by Theft which oCcurred during burial of the decedent.
D
2nd ioss: From fiood on December 29, 2020 of househoid furniture aiso not
compensated by insurance.
D
loss is not deductible and
2% loss is deductibie.
1 deductibie.
B. Both losses are not
estate 2019 leaving the foitowing data deductions
C. Both iosses are deductible from gross
an
74 Liza died on July 5.
not.
D. 1st loss is deductible and 2nd toss is P 40.000
Unpaid 2018 real estate taxes
Unpaid 2019 real property taxes 40,000
Income tax on income from Jan 1 to July 4, 2019 35,000
LOsses from fire that occured on July 3 (60% was
To be deductible, losses must occur after the death of the decedent but
compensated by insurance) 800,000
Casualty ioss on September 2019 450.000
within the period for the payment of the estate tax.
Building destroyed by earthquake on February 2020 1.300 000
72. Which stalement is iNCORRECT regarding "iosses"? Based on the above data the amount deductibie from gross estate is-

A P1365,000 P 1.865 000


Abuilding
that has been razed by fire immediately after the interment shall
still be included in the gross estate even if it does not exist 565.000 525.000
anymore at
the time of filing the estate tax return.
BIn a casualty loss, the value of the property is included in the gross estate
but subsequently deducted therefrom.
C i n a casualty loss, since
the value
of the property is induded in the
estate but subsequently deducted therefrom, it rmay be omitted in botn gross
tor Unpaid 2018 real estate taxes 0,000
after all there is no effect on the net Unpaid 2019 real propert taxes 40,000

Din estate taxation, the taxable esiate ineonie tax 35.000


amount of loss deductible is based on the value o Casualty loss on Septenber 450,000
the property lost
minus indemnity from an insurance
company Building destroyed by earthquake 300.000
Total deduction L,o5.0o
C The toss trom fire is not deductible because it ocrurred prior to the death
of the decedent, while the cost of the building destroyrd is dehuctible
To be deductible, the loss must take place after the because the fotuitous event oceurred within tihe perit for the paament
period the payment of the
for
estate tax.
death but witnn of estate tax.
In the computation of the
shall be included. However, gross estate, the market
the value of the value ot tne cted 5 Amount of claim against the debtor P 50 000
dimin
diminution from the emihcation
property lost snu hall be
from an insurance a Total assets of the debtor 500 000
deductible loss. company sha Tota liabilities of the debtor 800 c00
CPA REVIEWER IN TAXATION - AmpongaA
94 How much should be includedi the gross estate of the deceda
ESTATETAXATION - Multiple Choice Questions
A P 50,000
D
C. P 500,000
300,000
edent-cetk Computation of Total Net Estate:
95
B 800,000 Domestic shares
Foreign shares
P1,000,000
3.000.000
Tangible personal property, Phil 6.000.0000
A Total 10,000,000
Nonresideni aliens aretaxable on properties situated in the
Philippines
ciaim againstinsolvent person
is- only. Ifthere is reciprocity, intangible personal properties within are not
76 The deductible subject to estate tax.
C. P 300,000
A. P50,000
31,250 D 18,750 Claims against the estate, Linpaid mortgages. Claims against insolvents.
Unpaid taxes and iosses (CtCUL) are deductible but shal! be pro-rated
by applying the following fornula:

D Philippine gross estate


Total gross estate
x CUCUL
Amount of daim 50.000
Less: Collectible (500/800 x 50,000) 31259 78 Ta Pue, a norresident alien, singe, died leaving the following properties and
Uncollectible 18,750 deductions-

Stocks in a domestic corporation P 500,000


77. Che Cua, a nonresident alien, died leaving the following assets Stocks in a foreign corporation 500,000
Tangible personal property, Philippines 1,500,000
Domestic shares P1,000,000 Deductible expenses
Foreign shares 3,000,000 500,000
Tangible personal property, Philippines 6,000,000 Assuming there is no reciprocity, the estate tax due is-

Expenses (deductible) 1,200,000


AP1.100,000 C P103,000
Note- The country where she is a citizen snd resident does not imposi 56 000 D. 96,000
transfer tax on transmission of intangibies of Filipinos.
B
The net estate subject to tax in the Philippines is -

A. P5,280,000 C. P4,780,000 Shares, domestic corporation P 500,000


B. 3,800,000 D. 4,280,000 Tangible personal property L500.000
Gross estate 2,000,000
Less: Deductions
Expenses (20/25) x 50o.000 P400.000
C
Standard deduction 500.000 900.000
Net taxable estate 1,100,00o
Tangible personal property P 6,000,000
Rate of tax 6%
Less: Deductions b6,000
Estate tax due
Expenses (6/10 x 1,20o,000) P 720,000
Standard deduction 500.000
L,220.000

Net estate 4780.000


CPAREVIEWER INTAXATION- Amponga
96 ESTATE
TAXATION Multiple Choice Questions 97
Estate:
Total Gross
Computation
of
P 500,000 Properties- Philippines P 2,000,000
Shares, donmestic corporation Properties Abroad

Shares, foreign corporation 500,000 Keal property -Japan P2,400,000


Tangible personal property L5O0.000o Stocks-Japanese Corporation 600.000 3.000.000
Total
2-500,.000 Totai gross estate 5,000,000

information: Gross estate 2,000,000


based on the following
Numbers 79 and 80 are Less: Deductions
had the
residing in Munich, Germany the following data
fail..
German CUCUL
Mhar Dehrer, a Losses, abroad 40,000
death:
at the time of his Claims against the estate 25.000
Toial 65.000
Expenses. in the Philippines P 25,000
inaured
Funeral expenses (2/5 x 65,000) 26.000
Funeral expenses
incured abroad 55,000 Standard deduction 500,000o 526.000
ACCountant's fees and
audit fees 5,000 Taxable esiate ,474.000Q
-2 months before decedents death 50,0
Medical expenses
Losses on the property
located abroad 40,000 who died December 4, 2020 is
25,000 81. Which of the foilowing properties of Etang
Claims against the estate deduction?
subject to vanishing
Properties: three (3) years ago from Mitsubishi Motors
Osaka, Japan P2,400,000 Property 1-Car purchased
Real property iocated in Batangas City
Condo unit situated in Davao Cty 1,000,000
Shares of stock in Japanese Corporation 600 000 Property 2- Land inherited from her mother in 2017 the estate tax
1,000,000 thereon have not been paid.
Other tangible personal properties Phils
-

friend in 2018
79. The gross estate on the estate of Mhar Dehrer is Property 3- nter vivos donation from a

C. P 2,000,000 Property 4- Carpurchased with cash inherited December 2 2020


A. P5,000,000
B. 1,000,000 D 2,600,000 Property2Propeity2 Properny3 Property4
No No Yes Yes
No No Yes No
C Yes No No Yes
Yes Yes No No
Condo unit in Davao City P1,000,000
Other tangible properties 1000,002
Properties/gross estate - Philippines 2.000,000o
A

the decedent cither by donation or by


80. How much is the taxable estate? The car must have been acquired by
inheritance. Acquisition by onerous
title such as by purchase is not stubject
A. P1,935,000 C. P1,435.000 to vanishing deduction on the purchaser.
B. 1,774,000
the tax on the prior transfer
must
D. 1,474,000 To be entitled to a vanishing dcdtuction,
lhave bee paid.
D
98
CPAREVIEWER IN XATION-Ampongan TATETAXATION Multiple Choice Questions 99
Although Property 4
has been acquired by purchased, the
gratuitous title.
the money used .

The spouses shall be governed by the absolute


propety acquired by community of property
regime Nonetheless, the death of Cristita on May
20, 2020 will not
Subject her share in the land to a vanishing deduction
82 Baby Villanueva died leaving
a
propery which was inherited theaee years a The spouses shall be governed by the absoBute community
the vanishing deduction, which of D of
from his father. In computing
the following reaime. Thus, if Mendeil dies on May 20, 2020 only his one-haif property
tems shali serve as a
er deduction in computing the the land shall be subject to a vanishing deduction sharein
the initia! basis? deduction
A Benefits received under RA 4917
B Family home C
CStandard deduction
D. Transter for public purpose The spouses shall be governed by the absolute commtnity of
property
regine be cause ihe pre-nuptial agreenient was not reduced in witing.
The death of Cristita wili not subject the property to 2 vanishing
because it was Mendell, her husband. who acquiret the property by
deduction
D
gratuitou1s title.

83 Statement 1: Vanishing deducton 1s aways a deduction from the excissi 85 The foliowing condhtions must be complied in orderthat a vanishing deduction
properties of the decedent uSive
shall be allowed as deduction from gross estate, except
Statement 2: A property is allowed vanish1ng de uction if it has
been acouired The property can be identified as having been received from a donor or
thru exchange with a property inherited within 5 years prior to the death of the prior decedent.
present decedent. The property can be identified as having been acquired in exchange for
property so received from a donor or prior decedent.
A tatement 1 is true, Statement 2 is false
BStatement 1 is false; Statement 2 is true. The transfer tax on the previous transfer has been finaly detemined and
C. Both statements are true. pa:d
The prior transfer must have been either thru onerous or gratuitous
D. Both statements are faise.
transfer.

B
D
84 Christopher died on October 5, 2017 86. Which of the foilowing statements about "vanishing deduction is TRUE?
leaving a parcel of land
PB00,000 to his nephew, Mendell. On June 10, 2019, Mendellvalued mamee
a
Cristita Pnior to the celebration of A For a vanishing deduction to be deductible, the property must have
the marriage, they oraily agreed that ey formed part of the gross estaie situated in the Philippines of the prior
shai! be governed by the conjugal partnership of gains. ransferor.
as vanishing
Which statement is CORRECT? A family home of a non-resident alien maybe claimed
and the estate tax
deduction if it has been inherited within five (5) yearS
AIne spouses shall be governed by the on the previous transfer has been paid at
the time of death of the
Thus, if Mendelil dies on gjains
conjugal partnership ofshall be
May 20, 2020 the vanishing deduction S present decedent.
present
B. The
chargeable against his exclusive properties. Vanishing deduction maybe be
allowed on the estate of the
spouses shall be governed by the still alive.
absolute community of prop decedent even if ihe prior transferor is
reqime. Thus, if Cristita dies subje
on May 2020 the iand shal
vanishing deduction of one-half of its20,value.
d
to
CPA REVIEWERIN TAXATION- Multiple Choice Questions 101
100
deductibBe, there should
Ampongan ESTATE
TAXATION-

always be two The interval of time from the date of death of father to the date of death
alo
deduction to be
For a vanishing
D. five years from receipt of property of Rodolfo is 3 years, 1 inonth and 6 days, computed as follows:
deaths within
Year Monti Day
2020 5 10
2017 4
C
3 6
A vanishing
deduction may be alilowed even if the property was
provided dated
.

outside the Philippines during the prior transfer, present t it is


at the time of death of the Val Hallada died on November 20, 2018. Some of the properties he ieft are
Phiippines
situated in the
a0

not deductible eit


situated outside the Philippines is the following
A family home
or as a vanishing deduction because these two ita Market Value
a famiiy home t. Date 0eath of Val
that the property must be Situated within Mode of Date of
deduction require the
ACauisition Acquisition Acauired Halada
Philippines. Assets 7-3-14 P 500,000 P 350,000
Land Donation
In vanishing deduction, the irst transter of the property maybe inter Purchase 10-2-17 800,000 980,000
living at the time of death of Car
vivos. Thus, the prior transteror miay stili be a
two deaths within pern0d of five years is not Other information
the present decedent, and the decedent amounts to P3,000,000
deductibility. 1 The gross estate of
necessary for its for P50,000 when it was acquired and Val paid
2. The land was mortgaged
are not deductible from the gross estate of a the sanie before he died.
medical
87. Ail of the foilowing, except
one, deductions totai P125,000, which includes
3. The aliowable
nonresident alien excludes bequesi to a charitable institution in the
expenses of P30,000. t
C. Family home amount of P50,000
A. Funeral expenses
D. Standard deduction
B.Medical expensesS The vanishing deduction is (RPCPA)-
CP67,783
A P 58,100
D 67,083
57,500
D
are not deductible
Medical expenses, family home and funeral expenses
condition.
from the gross estate of a nonresident alien without A
fixed of P5o0,000. P 350,000
Standard deduction is aliowed at a amount
ower vale 50000
Less: Mortgage paid 300,000
88. Rodolfo, a citizen of the Philippines and resident of Bacolod City,
died testate Initial basis
inherited from nis ess: Deductions (pro-rated)
on May 10, 2020. Among his gross estate are properties 9.500
deduction wi DE L300,000/3,000,000 x (125,000-30,000)
deceased father who died April 4, 2017. What percentage of 290,500
used in computing the amount of vanishing deduction? (RPCPA) Base more than 5 years) 20%
not
Rate (more than 4 years; 58.100
A. 80% of the value taken as basis for vanishing deduction. Vanishing deduction been
have
B. 100% of the value taken as basis for vanishing deduction. the property must
To be subject to vanishing deduction, Those that were acquired
C. 60% of the value taken as basis for vanishing deduction. decedent gratuitously.
acquired by the present to said
deduction.
D. 40% of the value taken as basis for vanishing deduction. transfers are not subject
Dy onerous

D
TAXATION-
CPA REVIEWER IN Ampongan ESTATETAXATION=Multiple Choice Questions 103
102 Bequests to charitable institutions and medical expenses are not part of
items under CUCUL ar Value to take 550,000
Only the
the multiplier deductions.a r e included
if any, ded as part of said transfers
deductio) Less: Mortgage
Initial basis
paid on car 50.000
500,000
for public purpose,
of the followins
Less: Deductions (500,000/5,000,000)xP 700.000o 70.00Q

90 in determining
the net estate
of tne decedent, wnich
folilowing ruies is Base
Rate (more than 2 years, not more than 3 years)
430,000
corect? (RPCPA)
Vanishing deduction 258,000
is included in h e gross estate of a decedent who is
A Real estate abroad
a nonresident alien. If the same rate shall be applied to both properties, the vanishing
B Shares of stocks being ntangible property shall be included in the deduction shall be computed jointly for both properties.
wherever situated.
decedent's gross estate f different rates are applicable, the vanishing deduction shal! be
must be subject to limitations.
Vanishing deduction computed separately.
Funeral expenses are deducible to
the extent of 5% of the tota
gross
estate but not exceeding P200,000. 92. Elopre, married June 5, 2017 died on April29, 2019 with the folowing data:
Gross estate - community property, P3.000,000, exclusive, P2,000,000.
Said amount inciudes a land which he received as gift from his father a month
C before the marriage, valued at PS40,000. His father mortgaged the land for
P20,000 which was paid by Elopre Elopre mortgaged also said land for
91. Pepe died on August 15, 2019. His data are as follows P50,000 but was abte to pay onily P20,000 unti his deaih. Expenses claimed
(excluding the unpaid mortgage) amounted to P170,000.
Community properties P 2,000,000
Exciusive properties of Pepe 3,000,000 The vanishing deduction is
Exctusive properties ot Pepe's wife 1,000,000 P 383,800 P 384,000
Deductions (except standard deduction) 700,000 B. None 380,000
included in the P3,000,000 is a parcel of land worth P200,000 and a car
worth P400,000, respectiveiy.
The land was donated to him oy his uncle on May 4, 2017 with a vaiue
of P150,000. At the time of the donation, the land was motgaçed for P30,00 Value io take P540,00o
Less: Morigage paid (20,000 + 20.000) 40000
which was paid by his uncle. The car had a value of P500,000 when it was
mother 2 4 years ago and mortgaged for P50,000 Initial basis 500,000
nherited
which wasbypaid
Pepe from hisbefore Less: Deductions (pro-rated)
by Pepe he died.
170,000
Expenses claimed
The vanishing deduction on the estate of Pepe is -
Unpaid mortgage (50,000 20,000) 30.000
-

P 258,000 Tota 200,000o


C. P 283,800 500,00o/5.000,000) x 200,000 20.000
B. 262,520 D. None 480,000
Base
Rate (imore than 1 year, not more than 2 years) 80%
Vanishing deduction 384.000
A

Lower value of: 93. In Number 92 above, the net taxable estate is
Land
Car P 150,000
400.000
CPAREVIEWER IN TAXATIO ON-
104
A. P2,016,000
C. P2,208,000
Ampongan ESTATETAXATION -Multiple Choice Questions 105
D. The amount of
1,208,000 vanishing deduction is
A P190,000 C. 26,250
B 95,000 D 181,250
D
Community Exclusive Total
Gross estate 3,000,000 2,000,000 5.000,000 B
Less: Deductions
Car
Ordi nary
Amount cdaimed 170,000 Land
P 200,0000

30,000 Total lower value 500.000


Unpaid mortgage 700,000
Vanishing deduction 384.000 Less: Morlgage paid
Standard deduction (584,00o) Initial basiS 200.000
Net estate
5.000.00o) Less: Deductions (pro-rated) 500.000
Less: Share of surviving spouse Claims vs. estate P 30,000
Gross community 3,000,0o0 Bad debts
TPP
20,000
Less: Community expenses 584.000 50,000
Net community property 2.416,000o Mortgage (300,000-200,0000} 100.000
Share (2,416,000 /2) Tota 200,000
Net taxable estate
1208,000o (500.000/4,000,000 x 200,000)
Base 25.000
75,000
Rate (more than 4 years)
94. Alfonso, decedent, had the following data at the time of his death on Vanishing deduction
20%

2018: July 20 95000


95. Statement
Prior decedent -father Present decedent 1: ünpaid loans contracted
if not notarized if notarization of
prior to death may be deducted even
Car P 250,0000 P 200,000 contracts is not a business policy of the
Land creditor.
500,000 900,000
Statement 2: For estate tax purposes. several
The land had an unpaid
mortgage of P300,000 at the time it was family homes may be deducted
inherited from his father on January 5, 2014. provicied the maximum amount is P10,000,000.
Alfonso was abie to pay
P200,000 before his death. The gross estate of Alfonso was A. Oniy Statement 1 is correct.
while the expenses amount to P4,000,000 B. Both Statements are correct
P900,000, broken down as foilows C. Only Staiement 2 is
Actual funeral expenses
P175,0000 OBoth Statements
correct
Claims against the estate are incorrect.
Claims against insolvent 30,000
persons 20,000
Legacy to Cebu City for public purpose 50,000 A
Medical expenses
RA 4917 545,000
80,000 96. Which of the following statements is not correct?
tncluded in the gross estate is
his family home. It was purchas
on February 2016 at P1.8 million.
of P2,000,000. At the time of death, it had a
A The deduct le amount as Standard deduction is P5,000,000
vaiu B The deductible amount as Standard Deduction maybe P500,000
106 CPAREVIEWERINTAXATION-Amp ESTATETAXAIION-Multiple Choice Qiestions 107
deduction from gross estate
home is always an item of Ahedical expenses are deductible from the gross estate if
Family home should not exceed P10,000.C00 D. they have
D. The deductible family been incured in the Pnilippiries by a Hongkong national who visited the
Philippines as a tourist.

C
C
statements is FALSE relative to a famiy he.
me?
97 Which of the folowing

A decedent who was


mamied at the time of death may not i
have
og Statemeni 1: An unmatied individual cannot have a family home

deducton for family home. statement 2 The appiances and househoid furniure and fixtures are
The value of the family home must be inciuded in the gross estate an
B. included in the value of family home
to a maximum amcunt of P10 000 A
claimed as deduction therefrom
inctude the value of his family home in. 1 is comect.
A non-resident citizen shall
the A. Oniy Siatemant
ciaim SUch amount as a deduction therefrom Both Statements are corect.
gross estate but cannot B.
if the famly home is ailcwed a varishing deducion and a subject of
an C Only Statement 2 is corect.
unpaid morigage, the deductübie amouni snoutd be net of vanishi DBoth Statements are incorect.
shing
deduction and unpaid mortgage.

D
D
A family does not refer only to a spouse and children. It may also pertain tc
A decedent may not have a deduction for tamily home if he or the spouses brothers, 5isters and parents.
do not have such property. The term "family home" inciudes oniy the vaiue of the land and the house
The entire value of the family homeis inciuded in the gross estate if the thereon.
decedent was a resident or citizen of the Philippines.
To be deductibie, the family home must be situated in the Philippines. 100. Which of the foliowing siatements is FALSE?
However, the maximum allowable deduction is P10,000,0o0 0nly. A in estate taxation, the standard deduction from the gross estate is always
if the family home is subject to a vanishing deduction and an unpaid P5,000 000, whether the decedent was married or not
mortgage, the amount deductible shall not be diminished by the arnount B if the decedent was a non-resident alien, the standard deduction shall be
of the vanishing deduction and unpaid mortgage. pro-rated according to the ratio of the Philippine gross estate over the
total gross estate.
C A standard deduction is an item of deduction from gross estate which
6 Which of the foltowing statements is TRUE about "medical expenses?
does not diminish the distnbutable estate even if it diminishes the taxable
A Hospital expenses are deductible only from gross estate if unpaid at the estate
Benefits received under RA 4917 must be included in the gross estate irn
time of the death of the decedent even if it has already been paid at the
time of
D
filing the estatetax retum. order to be deductible therefrom.
B Hospital bills which have not yet been paid at the tirme of deaui
deductible from gross estate as claims
C.
against the estate
Medical expenses are not deductíble from the
gross estate eveil B
Constitutes hospital bills incured on the decedent.
A standard deduction is deductible from the gross estate of a non-resident
alien at a fixed amount of P500,000; it is not also deductible in computing
CPA REVIEWER IN TAXATION- Am. ESTATETAXATION-- Multiple Choice Questions
108
of the citizenship or
npongan 109
the net distributable estate regardess residence of the
or resid.
A. P 1,900,000
1,600,000
C. P 3,100,000
D. 2,800,000
decedent. B.

Australian, died. if
the decedent was a resident.of Manila, how
101.Dasing Jose,deductible standard deduction? B
much is the

P5,000,000 C P1,000,000
None
House P1,500,000
500,000 Land 400.000
B.
Total 1,900,00b0
Less: Reimbursed amount 300.000
Family home 1,600,000
A
the deduCtible amount of famity home is-
information, if the decedent was a resident of Aishrat 05. Before iquidation,
In the preceding
102.standard he C. P 800,000
deduction is A P 1,000,000
B. 950,000 D 1,150,000
P5,000,000
C. P1,000,000
A D. None
500,000
D

House, conjugai (1,500,000/2) 750,000


B Land. exclusive 400.000
Deducible 1,150.00
tems 103 to 107 are based on
the following information
value of the family home
Decedent died leaving a famiy home composed
of the following: House 106. Under absolute community of property regime, the
the spouses worth P1,500,000, and the iand in which IS
owned in common by
at P400,000. At the ime the house was C. P1,500,000
he exclusively owned valued A. P 1,900,000
cost of P300,000. They also own a vacation house in D. 3,100,000
constructed, it had a B. 1,600,000
Baguio worth P1,200,000.
the famiiy home is classified
103. if under conjugal partnership, after its liquidation A
as a
1,500,000
House, community
A. Conjugal property Land. exclusive 400.0002
Exclusive property of the decedent
3 Total
1900,000
C. Excusive property of the surviving spouse
D Partly conjugal and partly exclusive of the decedent the deductible amount of
107. Under absolute community of property regime,
family hone is-
C. P 1,900,000
A. P 1,000,000
A B 1,500,0000 D 1,150,000
home
of the tamily
104.
Under conjugal partnership, the amount of conijugal portion
after liquidation is
110

A
CPA REVIEWER INTA
XATIONAmpongan ESTATE TAXATION- Multiple Choice Questi
110.
The vanishing
deduction is-
111

House, community (1,500,000/2) 750,000 A. P1,786,056.52 C. P1,773,708 20


400,000 1,842,045.76 D. None
Land, excdusive B.
1,150,000
Deductible

through 110 are based on the following information:


Questions 108
Lower value/ Initial asis P2,600,00o.00
or tne PhHippines residing in a n .
108. Mama Mathay, widow, a citizen 2019 leaving the following ancouver
properties Less: Dedictions (pro-rated)
Canada, died on December 20, 2.600.000/7.430,000 x P 850,000) 297.442.80
from her husband on May 3, 2,302,557.20
Real property (inherited Base
Rate (more than 1 year, not more than 2 years) 80%
2018 valued then at P2,600,000) P2,960,000
Personal properties
in Canada 1,300,000 Vanishing deduction 1,842,045.76
properties in the Philippines 670,000
Real and personai
Family home in
Canada 2,500,000 111. Decedent died ieaving a famiiy home composed of the following: House,

worth Peo0,000, and the land in which he exclusiveiy


Cbligations: conjugal property
vacation house in Baguio worth
Funeral expeNsES incuITed in Canada 250,000 owned vaiued at P400,000. He aiso owns a
Other deductible expenses 850,000 P700 00O.

The gross estate of Mama Mathay is


-

The deductibla amount of family home

C. P6,760,000 A. P800.000 C. P1,900,000


A P7,430,000 D1,00,000
670.000 8 1,20C 000
7,070 CO0

A A
House, conjugal (3o0,000 x 2 P 400.000
Real property P2,960,000
1,300,000 iLand, exelusive 400,0002
Personal properties, Canada 800,000
Real and personal properties 670,000 Deductivle
2.500.0009
Family home, Canada information.
Gross estate 7.430,000 Questions 112 and 113 are based on the following
and under
A reSIdent alien decedent was married at
the time of death in 2018
109 The deduction for family home is regime. Among the properties
tne system of absolute community of property
estate was the family home consisting of
t tne gross
A. P2.500.000 C. P1.250,000
market value P1,000,000
1,000,000 D. None Land, inherited duning the mariage, 8,000,000
inherited land
f1ouse built by the spouses on the

The deduction for family home is


D P1,000,000
A. P9 000 000 C.
8,000,C00
amily home which is situated outside the Philippine 5,000,0000 D.
deductible from gros estate.
112 CPA REVIEWER IN TAXATION -Ampor ESTATE TAXATION- Multiple Choice Questions 113
L B D

Land, excdusive 1,000,00o Gross estate Deductible


House, community (8,000.000/2) 4.000.000
Land. exclusive P2.000,000 P2,000,000
Deductible family home 5.000,00o House, cOmmunity 3.000.000 1500.000
Total 5,009,009 3.500.000
113 How about if the decedent was a non-resident citizen?
G How about if the land is an eKCJSIVe property of ihe surviving spouse whiie
A. 9,000,0000 P1,000,000 the house
!s conjugat propety cf the spouses?
5.000,000 None
Gross estate Deduction Gross estate educion
P5,000 000 P5,000,000 CP3.000.c00 P1 500 000
3,000.000 2 500,000 3 CO0 000 3,000.000
D B

The family home of a nonresident is


situated abroad. To be allowed as
deduction from gross estate, the family home must be situated C
in the
Philippines. Gross estate Deducihle
Land, exclusive of spOUse None None
Questions 114 through 117 are based on the folowing information: House. conjugal P3.000.000 P1.500.000
114 Sonny, marned, died leaving a family home consisting of land valued Total 3.000,00 L500,000
P2.000,000 and a house, P3,000,000 How much is includible in at
deductible from gross estate if both the land and the house and of the spouses
are community 117 How about if the land and the house are Common property
properties? who are both Amencan citzens residing in Canada?

Gross estate Deduction Gross estate Gross estate Deduction Grossestate Deducion
. P5,000,000 P3,500,000 C P2,500,000
Deduction P5,000 000 P5.000, 000 C None P2 500 00o
B. 5,000,000 2,500,000 P2,500,000 5 000 000 2.500,000 D None None
D. 5,000,000 3,500,000 B

B
D
taxable that are situated in the
only on properties
Nonresident aliens are
Land, community Gross estate Deductible nonresident alien is situated
P2,000,000 P1,000,000 Phiippnes. Since the family home of a

House, community trom gross estate.


abroad, it is not taxable and not also deductible
Total 3.000,000 1500.000
5.000.000 2,500.000 home valued at P1500 0co
0
decedent, marned aied leaving family
e a
115. How about if the and the lot (exciusive property)
land is exclusive property of the decedent while the houSE Gomposed of the house (conjugal property)
community? value of the family
home pertains to the
Seventy percent (70%) of the
Gross estate Deduction Ouse, while pertains to the lot
thirty percent (30%)
P5,000,000 P5,000,000 C Gross estate Deduction The amount deductible from gross
estate is-
8. 5,000,000 P2,500,000 P2,500,000
2,500.0000 D.
5,000,000 3,500,000 A P 1.500,000
CP975.000

525 000
B 1,000000
114 CPA REVIEWER IN TAXATION TION- Ampongan rATE TAXATION-Multiple Choice Questions 115
Conjugal Exclusive Total
C
Real properties P3,000,000
2
House (Pi,500,000 70%)
x x
P525,00o Family house 1,000,000
Lot (Pi,500,000 x 30%) 450.000 Other real properties P2,000,00o0
2.000.000Q
Amount deductible 975.000 Family lot
4,000,000 4,000,000 P8,000,000
Gross estate
Less: Deductions
Bong, singie and a resident citizen, died With properties constit di
119 estate of P6,000,000. Actuai funeral expenses amounted to Pa rogs Ordinary
Taxes and losses 300,000)
other charges against the estate amounted to P210.000, The estate tax duedu Special
IS Standard deduction 5,000,000)
C. P790,000 Fainily home
P 940,000 500,000
iouse (1,000,000/2)
B 56,400 D 47,400 2.000,000 (2.500.000)
Lo 200,000
Net estate
(4.000,000-300.000) x 2 L850,000
Share of surviving spouse
D -

Net taxabie estate

partnership of gains, the vanishing deduction


Gross estate 1: Under the conjugaB
P6,000,000 121. Statement exciusive properties.
from
Less: Deductions always a
is
deduction

the vanishing
Other charges P 210,000 absoiute community of property regime,
Statemet 2: Under the
Standard deduction 5.000.000 5210.0Q0 deduction
ded:cted also
maybe
frorm community property.
Net taxable estate 790,000
Rate of tax . False, False
6% A. True, True
Estate tax due False, True
47.400 B True, Faise

120. Decedent, marnied in 1985 without


the following:
an ante-nuptial agreement, died ieaVIg
A
Real properties P 3,000 000
Family house relates to Carl, married two (2) years ago, died leaving
1,000,000 122. The following data
Other real properties, exdusive of decedent 2,000,000 the following
Family lot, exclusive of decedent 2,000,0000
P14,000,000
Funeral expenses Gross estate
275,000 3 % years ago
Medical expenses Land donated by his father 200,000
Taxes and losses 650,000 Market value, date of donation 300,000
300,000 Market value, date of death 35,000
The net taxable
estate is -
Funerai expenses 15,000
A. P 2,450,000 Judicial expenses 100,000
B C. P 2,250,000 on land at the
time of donation
1,150,000 D. Unpaid mortgage 10,000
None Unpaid taxes 25,000
Losses 35 000
D ransfer for public purposes
116 CPA REVIEWER IN ION- Ampongan
ESTATETAXATION=Multiple ChoieeQuestions 117
Medical expenses 45,000 Unpaid mortgage P 40,000
of the land a Unpaid taxes
Cari paid P60,000 to the mortgagee before his death
year beforai 10,000
Losses 25,000
Assuming Cart was under conjugal partnership of gaine
the total Vanishing deduetion
55.560
exclusive property is
ordinary deductions from Total ordinary deductions from community property 130.569
A P75,000 C P 124,600 The unpaid mortgage is an obligation of the community property
B 55,560 D 130,560 because it is considered as an ante-nuptial debt which have redounded
to the benefit of the family.
The vanishing deduction is a deduction from the community property
D because the land is dassitied as a community property.

Ordinary deductions from exclusive property:


124.Alladin, filipino, married, died January 1, 2019, leaving the following properties:
Unpaid mortgage on land (100,000-60,000) P40,000
inhented from his brother who died May 3, 2017
Vanishing deduction 55.560 Riceland P 1,000,000
Transfer for public use 35.000
Total 130,560
Residential iand 2,000,000
inherited from his mother who died April 12, 2015 or five days after his
Computation ofvanishing deduction: marriage.
Value date of donation (lower value) Coconut land 420.000
P200.000o
Less: Mortgage paid Acquired thru Alladin's wife's labor
60,000
Initiai basis 140,000 Famiy home 2.000,000
Less: Deductions (pro-rated) Car 500,000
Unpaid mortgage (100,000 60.000) P40,000 Commercial land 1,000,000
Unpaid taxes 10,000 Gold necklace (acquired by Aliadin during a previous
Losses 25,000 marnage which had a legitimate descendant) 80.000
Transfer for public use 35.000
Total 110,000 The riceiand and the residential land were previously mortgaged for
P350,000 when inherited where P200,000 was paid by Alladin during
(140,000/14,000,000 x 110,000) 1,100 his lifetime
Base
Rate 138,900 The coconut land was mortgaged for P94,000 of which P14,000
40%
Vanishing deduction S5560 was paid before his death. Also Alladin, by wil, bequeathed to Marikina
City the sum of P200 000 for exclusively public purpose.
123 in Problem 122 above,
assuming that Cari was under The estate incurred the following expenses
property regime, the total amount deductible from absolute communy
the community propi Funeral expenses P 140,000
Judicial expenses 80,000
A. P 130,560
B 134,600 P174,600 Porticn of family home destroyed by fire on Jan 5, 2019 100,000
None Medical expenses 40,000
he gross estate of Alladin is
A A. P3,500,000 C. P5,250,000
B 7,000,000 D 3,957,020
118 CPA REVIEWER IN TAXATION- An. ESTATETAXATION- Multiple Choice Questions 119
VD 2:On coconut land:
B
Value to take P 420,000o
Community Exclusive Total Less: Mortgage paid
Family home P 2,000,000 Initial basis
14.000
500,000
406,000
Car Deductions (406,000/7,000,000 x 530,000)
Commercial land 1,000,000 30.740
Base 375,260
Riceland P1,000,000 Rate (more than 3 years; not more than 4 years) 40%
Residential land 2,000,00o Vanishing deduction
-150.104
80,000
Necklace
Coconut land 420.000 VD 1 2,070.400
Gross estate 3-500,000o 3.500,000 P 7.000.000 VD 2 150,104

Ailadin was under absolute community ot property regime becaise Total 2.220.504
marriage took place on April 7, 2015 {nve (5) days before the deatth of 124 above, the net taxable estate is-
his inotherj. 126, In Number
Mariages celebrated on or after August 3. 1988 are governed by the A. P2,109,840 C. P515.000
absolute community of property regime uniess there was an ante B. None D 535.000
nuptial agreement between the due that they sfhail be govermed bya
diflereri regime of pmperty relationship.
B
125. in Probiem 124 above, the vanishing deduction is -

Communiy Excusive Total


A. P2,032,00 CP2.220,504 Gross estate P 7,000,000

8. 440 D. 2,070,400 Deductions


Ordinary
Losses P 100,00o
C UM. coeonut land 80.000
UM, riceresidential land P 150,000
VD1:On riceland and residential land: Donation to Marikina 200.000
Riceland P1,000,000 Vanisiing deductions 2,220.504
Residential land 2000,00 Tota 180,000 2,570.504 (2.750.504)
Vadue to take 3.000,000 Special -
Less: Mortgage paid 2) (1.000,00o)
200.000 Family home (P2,o00,000 x

Initial basis 2,800,000 Standard deduction (5.000.000)


Deductions (pro-rated) Net estate
Losses 1.660000
P 100,000 SSSS (3.500.000-180.000) x
Unpaid mortgage (94.000-14,00o) 80,000 Net taxable estate
Unpaid mortgage (350,000-200.000) 150,o00 on the coconut
land is a deduction from the
Donation to Marikina City The unpaid mortgage that the amount on the
200.000 because it is presumed
Total community property of the spouses.
530,000 benefited the community property
mortgage had
Base (2,800,000/7,000,000x530.000) 212.000
The unpaid mortgage on the
riceland and residential land is chargeable
brother
2,588.000 it was contracted by the
Rate (more than 1 year; not exclusive properties because
more than 2
years) 80% against
Vanishing deduction 2,070,400
and not by Alladin.
120 CPA REVIEWER IN TAXATION-
Ampongan ESTATETAXATION=Multiple Choice Questions 121
127. Lucy died leaving the following: Unpaic mortgage on riceland with a value of P1,000,000
Exciusive properties insolvent 200,000
Claims against Dimalupig, 35,000
Conjugal properties P4,000,000 the abov informatior
Judicial expenses 2,500,000 Based
Abandonado is
on the value of the
gross estate of
Funeral expenses 45,000
Notes payables (oniy % is notarized) 150,000 A. P 5,915,000 C. P4,685,000
Claims against insoivent persons (50% is
collectibie) 100,000 4,835,000 5,730,000
Proceeds of life insurance (berneficiary is
wife-revocable) 120,000
Death berefts under RA 4917 200,000
Medicai expenses (1/2 is not supported by
receipts) 180,000 A
The net taxabie estate is- 550,000
Family home P 2,230,000
A P1,892,500 C. P 1,520,000 Personal properties 2,500,000
265,000 D 1,862,500 RA 4917 150,000
Riceiand 1,000,000
Claims against insolvent peson 35.000
B Gross estate 5,915.000

Excusive properties
Conjuga Exdusive Total Numbers 129 through 132 are based on the fotiowing information:
4.000,00G
Conjugal properties5 2,500,000 On October 15, 2019, Benjamin, a filipino citizen and a resident of Manila
Ciaim
against an insolvent died intestate leaving his wie Diana and histwo ilegitimate children, Aubrey and
120,000
Proceeds of insurance Barbara. The estate of the deceased consisted of the following
200,000
Death benefits under RA 4917
180,000
Gross estate
3,0n0,000 4.000,000 7,000,000 ReaiHous
property conjugal
and lot (family home) - Manila. This property has an assessor's value
iess: Deductions
of P2.500,000 at the time of death but valued in the zonal valuation of the BIR for
Ordinary P3,900,000
Claims against the estate (1/2)
Bad debts (50%) 50,000
60.000 110.00) Personal property- conjuga
Special
RA 4917 Ihe total value was placed at P3,600,000
Standard deduction 180,00o)
5.000,0O0) included in the P3.600, 000 are proceeds of an irrevocable lifeinsurance policy
Net estate of P100,000 from Phil-Am Life insurance Company taken by Benjamin with Barpara
Less: Share 1,710,000
of surviving spouse (3,.000,000- 110,000) xY2
Net taxabie estate 1.445.000 Gs t e Dereficiary The premiums were paid out of conjugal property oft
he

265,000 spouses.
128. The aiso inctudes bank deposit with BPI in the amount of P600,000.
following data relates to the estate
of Abandonado
House and lot ne following deductions were claimed by the heirs
(family home) in Quezon City, zonal
value (assessor's P100,000
Personal properties value,
a. Funeral expenses
P1,150.,000) P2,230,000
b. Unpaid loaris, notarized 75,000
Benefits received from 2,500,000 ofthe estate 25,000
LOsses incurred during the settlement
his death employer as a consequence of
150,000
ESTATETAXATION=Multiple
ChoiceQuestions
122 CPA REVIEWER IN TAXATION- Amne 123
the bank deposit has been withdrawn ahead of the fiing of the
net estate if
is
of Benjamin
129 The total gross estate C. P6,400,000 132
The
he

return?

P 7,500,000
7,400,000
D. 6,100,000 A. P 100,000 C. P 250,000
350,000 D. None
B.

B
D
Number 131 below.
See the solution in Family home P3.900,o0o
home is Personal property (3.500,000- 600.000) 2,900,000
130. The deductible
amount of family Gross estate / gross conjugaB 6,800,000
P 1,000,000 Less: Deductions
A. P1,950,000
D. None Ordinary 100,0oo)
B 1,450,000
Special
Family home (1,950,00o)
Standard deduction (5.000000)
A
Net estate
See the solution in Number 131 below.
133. The tax due in the preceding question is
131. The net taxable estate is
AP15,000 C.P 250.000
A. P 100,000 C. P950,000 B. 36,000 D. None
B. 350,000 D. None

B
D
Amount withdrawn P600,00oo
Fainily home P 3.900,000 Tax rate 6%
Personal properties (P3.600,0o0 -

100,000) 3.500.000 Final withholding tax 36.000


Gross estate/ gross conjugal 7.400,000
Less: Deductions 134. Which of the following deductions cannot be claimed by a nonresident alien?
Ordinary situated in the Philippines
Claims against the estate .Vanishing deduction on a property
P75.000
Losses . Funeral expenses incurred abroad.
25.000 100,000) Family home situated abroad
Special I. Donation of a property for use by a foreign government.
Family home (P3,900,000 x 2) (1,950,0o00)
Standard deduction (5.000.000)
Net estate A. Ionly C. Ill and IV
3.50,000
Less: Share of B. and ! D. 11, I and IV
surviving spouse
Gross conjugal
7,400,00o
Less: Conjugal deductions L20.000
Net conjugal D
Share 7:300,0o0
(7.300,000 x 2)
Net taxable estate 3.650,0o0
124 CPAREVIEWER IN TAXATION- -Ampongan STATE
TAXATION= Multiple Choice Questions 125
based on the following information
Number 134 through 136 are Conjugal Excdusive Total
Fanily home
He died intestate on October 10, P1,900,000
Wilson died of a cara accident.
son.
2018, survived
201
Other personal properties 800,000
his wife, Ging and
by Other real properties
1,500,000
Exclusive properties of Ging: P 500,0O0
P 400,000
Cash
Receivable from PlICPA
Car 50,000
Lot in Quezon City ReceivalbBe from insurance
Other real and personal properties
2,000.000 Claims against insolvent
150,000
-30,000
800,000 Gross estate 730,000 4.200,O00 4.930,000
Exclusive properties of Wilson:
House and iot in Sta. Rosa, Laguna, family home
1,900,000
Other personal properties 136. The deductible share of surviving spouse is

Other real propertiesS


800,000
1,500,000 A. P 232 500 . P 112,500
Conjugal properties of the spouses: B 127,500 D 2.227,500
Cash in bank
Receivable as prize in a raffle sponsored by PICPA
500,000
50.000
Receivable from an insurance company where the son,
Gino was designated as a revocable beneficiary. The See the solution in Number 137 below.
premiums were paid out of the conjugal funds. 150,000
The following deductions were claimed' 137. The net texable estate is
-

Funeral expenses 195,000 A P1,962,500 CP1.850.000


Judicial expenses 15,000 B D. None
Claims against the estate, not notarized
1.977500
50,000
Claims against insolvent persons 30,000
Unpaid mortgage on other reai properties (contracted D
for the benefit of the conjugal properties) 200,000
Unpaid mnortgage on house and lot in Laguna (the Gross estate 4.930,00o
proceeds of which did not redound to the benefit of 350,000 Less: Dediictiois
the family) Ordinaryy
Accrued income taxes 35,000 Bad debts 30,000o
income tax on income eamed from October 11 to Unpaid mortgage-real properties 200,000
December 31, 20188 7,500 Unpaid mortgage, iagnna 350.000
Acerued taxes 35.000
135 The gross estate is 'Totals 265.000 350.000 ( 615.000)
A. P4,930,000 Speciat -
C. P4,850,000
B 4,900,000D amily home (1,900,00O)
D 8,130,000 Standard deduction 5.000,00)
Net estate
265,000} X 2 232,500
Less: Share of surviving spouse (730,000
Net taxable estate
126 ON- AAmpongan
CPAREVIEWER IN TAXATION-
ESTATE tAXATION- Multiple Choice Questions
Since the mortgage
on
real properties was contracted forsthe 127
family, the same
is considered as a ded from conjugalbenefit
partnership
tof the D
2 &3, Family Code).
ofthe spouses (Art. 121, par Bad debts
P 100,000
citizen, single Dit nead
of tamily, died Januan. Unpaid nortgage on family home 30,000
138. Alanis, resident
a Family home
foliowing are his data The Deductible 1,100.000
1,230,000
Properties: The house in Australia is not subject to
Real properties (net of family home of P1,100,000) P3,200,000 vanishing deduction because the
property is situated outside the Philippines. To be
House and iot in Sydney, Australia 1,500,00o subject to this
Other perSonal properties
deductiori, the property "must tor+n part of the gross estate situated in
800,000 the Philippines.
Deductions:
Funeral expenses 120,000 A0 Trito, a resident of uEzon City, died on June 5, 2019 with the following
Claims against insolvent persons data
100,000 Property acquired by Trillo before marriage
Claims against the estate, not notarized 50,000 P 1,500,000
Property acquired by his Wife before marriage
Unpaid mortgage on the family home 30,000 1,000,000
Famiiy house and lot, uezon City, certified
by barangay
ThepersonaB properties do not inciude shares of stocks
vaiued at
chairman, exciusive propety of Trilio
1,600,000
P50,000 which were purchased by the dececent írom Astra House in Marbe! City {exclusive of Trilto). certified as
Company one famity home by barangay captain
month prior to his death.
Proceeds of iife insurance, jrrevocable, beneficiary is tne
3,000,000
The house in Sydney was inherited by Aianis from his father who died estate
2 % years ago. Said property was nortgaged for P200,000 which was 500 000
paid Ciaims against insolvent debtors (40%
by the decedent before his death uncoliectibie) 100 000
Inter vivos donaions to City Government of Quezon
The gross estate is - Actual funeral expenses ({50% paid by
200,000
relatives) 300,000
A. P4,050,000 Judicia! expenses
C. P6,650,000 250,000
B. 6,750,000 The net taxable estate under
D. 5,550 000 conjugai partnership is-
A. P6,040.000
C. P1,550,000
B 1,580,000 D None

Real properties
P 3.200,000 D
Family home
House and lot, 1,100,000
Austraiia
Other personal properties 1,500,000 Conjugal Fxcusive Total
Claims against insolvent 800,000 Acquired betore marriage P1,500.000
Shares of stocks persons 100,000O Fanily home, Manila 1,t00,0O0
Gross estate 50,000 House in MarbeB City 3,000,0)0
6.750,000 Proceeds of insurance P500,0000
139. Claims against insolvent
The total deductions 100.000
A. P 250,000 (excluding standard deduction) IS Gross estate 600.000 6,100,000 P6,700,000
Less: Deductions
B 300,000 C. P1,250,000
D.
Ordinary
1,230,000 Bad debt (100,000 x 40%) 40,000)
ESTATETAXATION-Multiple Choice Questi
128 CPAREVIEWERIN TAXATION-Amnom 129
Special
pongan ltems
449 through 144 are based on the following information:
Family home nanduko married 2013, died on August 30, 2018. He left
Standard deduction (1,600,00o) properties and obigations:
the foilowing
Net estate 5.000.000)
Less: Share of surviving spouse 60,000 Properties:

Cash in bank
Gross conjugal P 200,000
Less: Conjuga expenses
600,000 Residential lot inherited from his father on June 12, 2015
1,200,0000
40.000 Famiy home:
Net conjugal
Share (560,000 x 2)
560,000 House (communty property) 1,300,000
Net taxable estate 280.000 Lot (exclusive property of Penduko) 1,G00,0000
Personal properties acqured by the spousES during marriage 200,000
The house in Quezon City is the true fainiiy home because it is the place Receivabie from his Sister (insolvent) 100,000
where the famiBy resides. Inter vivos donatien from his mother on July 2018, revocable 150,000
Receivabie from SSS as indemnity for hospitalization 12,000
141 The net taxable estate under community property regime is- Obligations:
A. P6,040,000 Unpaid mortgage on the residential iot contracted by the father
C. P1,550,000
B 1,580,000 D. None At the time of death of father 300,000
At the time of death of Penduko 100.000
Funeral expenses (40% were shouldered by relatives 80,000
Judicial expenses (30%% were incured after 6 months) 35,000
D
Claims against the estate (includes unpaid medical expenses
of P12,000) 35.000
Acquired by Trillo
Cominunal Exciusive Total Unpaid nortgage on the house (loaned to Penduko's sister) 100,000
P1,500,000
Acquired by wife 1,000,000 Casualty loss (50%o was indemnified by the insurance company) 60,000
Family home, Manila Donation to Barangay Engkantao (verbai donation) 25,000
House in Marbel City P1,600,0oo
3,000,00o estate of Penduko is
Proceeds of insurance 500,000 142 The gross estate on the -

Claims against insoivent A. P 4,062,000 P 3,962,000


100.000
Gross estate B. 4,000,000
Less: Deductions 3,100,000 4,600,000 P7.700,000 3,900,000o
Ordinary
Bad debt
40,000) D
Special
Family home (1,600,00o) Excdusive Community Total
Standard deduction 5.000.000 P 200,000
Net estate Cash
1,060,000 Residential lot P1,200,00oc
Less: Share of
surviving
Gross communal spouse Family house 1,300,0000

Less: Communal expenses 3,100,00o Family lot 1,000,00o


40,000 200,000
Net conjugal Personal properties
3,060,000 100.000
Share (3,060,000 x 2) Receivable from sister 2.200,000 1,800,000 P4,000,000
Net taxable estate 1.530.00
Gross estate
130 CPA REVIEWER IN
The property subject
ATION-Ampongan
the revocable donation made by th
of
ESTA TACETAXATION- Multiple ChoiceQuestion
Unpaid mortgage on the house
100,000
131
although physically transferred Penduko does not transfer ti.
to
actually an inter vivos donation subject to estate to
mothim
her Casualty loss
Vanishing de uction 30,000
364.700
because it is on
mother. the Total 717.700
Beheits received from SSS and GSIS are exempt from estate tax. Donations for public purpose must be testamentary in character. Oral
The claie against Penduko's sister is incuded already in the gross e donations are not valid.

as a "Receivable from his sister." estate The unpaid mortgage on the lot is chargeatble against the exclusive
properties.

143. The vanishing deduction on the estate.of Penduko is-

P 357,450 P 300,500o sA5 The estate of a non-resident,


deduction
not
citizen of the Philippines is allowed to clain
from gross estate
B. 375,540 364,700 this item as
A. Funeral experises Medical expenses
B. Judicial e x p e n s e s D. Standard deduction

Value of property P1,200,000 D


Less: Mortgage paid (300,000 100,000) 200.000
Initial basis 1,000,000 are deductible in the net
146. All of the following items, except one computing
Less: Deductions (pro-rated) nonresident alien
Unpaid mortgage on lot taxable estate of a
P100,0o0
Payable ofthe estate (35.000- 12,000) 23,000 A. Vanishing deduction
Unpaid mortgage on house 100,00O B. Share of the surviving spouse in the community property
Bad debts loan to sister) 100,00o C Transfer for public use

Casualty loss (60,000 x 50%) 30,000 D Funera! expenses incumed in the Philippines
Total 353.000
(1,000,000/4,000,000 x 353,000) 88.250
Base 911,750
D
Rate (more than 3 years not more than 4 years) 40%
Vanishing deduction 364,700 Questions 147 and 143 are based on the following information
his home
sident aiien who died in the Philippines but buried
in
144, The total ordinary deductions from the community property of Penduko is A nonre

A
country. had the following data on his estate
P 807,9500 P 782,9500 P 3,000,000
B. 717,700 682.950 Properties, Philippines 6 000 00o
Properties, foreign country
200 000
Funeral expenses, Philippines 300,000
B Funeral expenses, abroad
80.000
Judicial expenses, Philippines
100,000
Ordinary Deductions: Judicial expenses, abroad
74,000
Bad debts Unpaid taxes, Philippines
P100,00o 100,000
Unpaid mortgage on residential lot 100,000 unpaid mortgage on property in the Phiippines
160.000
Payable of the estate Unpaid mortgage o11 property abroad
23,000
TAXATIO Multiple Choice Questions
132 CPA REVIEWERIN
Claims against the estate, abroad
XATIONAmpongan
300,000
ESTATE
Unpaid taxes
Unpaid medical expenses
12,500
133

Estate tax paid abroad 120,000 550,000


is the net estate?
estate? How much
147. How much is the net taxable
P 3,268.500 C. P 3,238,000
P 2,282,000 A.
A. P2,342,000 D.
2,215,0000 B.
3,278,750 637,500
B 2,526,000

D
A
Exclusive Community Total
Gross estate, Philippines P3,000,0oo Personal properties P2,500,000 P1,750,00o
Less: Deductions Family home 2,000,00o
Unpaid taxes, Philippines P 74.000 Real properties 1400.000
Gross estate 4:500,000 3.150.000 P 7,650,000
Unpaid mortgage, Philippines 100,000
Ordinary deductions:
Claims against the estate, abroad 300.000
Total 474,000 Unpaid taxes 12,500)
Special deducticns:
(3.000,00o/9.000,000 x 474.000) 58.000
Standard deduction
Family home (2,000,000)
500.000 658.000 Starndard deduction 5.000.00o)
Net taxable estate 2342.00 Net estate 637.500
- Share of surviving spouse (3,15o.000 12.500)/2 1,568.750
148. The estate tax payable is -
Net taxable esiate
P 20,520 C. P 140,520
93,680 D. 136,920 150. The net distributable estate is

A. P 3,278,750 C. P5,483,750
C B 5,683,750 5,418,750

Net taxable estate 2,342,00o


Rate of tax B
6%
Estate tax due and payable 140.520o Exclusive Community Total
Personai properties 2.500,000 750,000
149. Eleanor, resident citizen, married and under the absolute Family home 2,000,000
property regime,
commury
died on August 20, 2019. The following are the daia Real properties L400.000
properties and obligations:
o Gross estate 4.500,000 3.150,000 7,650,000

Exclusive properties of Deductions:


Personal properties
Eleanor Funeral expenses 220,000
P2,500,000 Unpaid taxes 12,500
Family home 550.000 782.500)
2,000,000 Medical expenses
Community properties: Net estate 6,867,500o
Real properties 1,183.750
1,400,0000 782,500)/ 2
Personal properties Share of surviving spouse (3.150,000 5.683.750
Funeral expenses, actual 1,750,000 Net distributable estate
220,000
134 CPA REVIEWER IN TAXATION- Ampono ESTATE TAXATION=Multiple Choice Questions
ESTA:
135
A. P155,000

147,000
C.P
D.
72,000
110,000
B.
C. TAX CREDIT & ADMINISTRATIVE
PROVISIONS
Net estate, Philippines
P1,200,000
Net estate. United States 1800.000
Taxable estate
3,000,000
Rate of tax 6%
151. One is not entutied to tax credit for estate taxes paid to foreign country Estate tax 180,000
A. Resident citizen Resident aien Less: Tax credit
B Nonresident citüizen Nonresident alien Tax paid in U.S. P 25.000
Limit (18/30x180,000) 203.c00
Credit ailowed (lower) 25.000
Estate tax due after tax credit 55.900

152. entited to tax credit lterms 155 and.156 are based on the following iniormation:
Ali of the foliowing, except one, are on estate tax paid in
Lana, singie, an american residing in the Philippines died ieaving the
foreign country.
A Nonresident, not citizen Resident alien foiiowing properties
B. Nonresident citizen Resident citizen LOcation of prOperty Netestate Estate tex paid
Phiippines P 1,500.000
"A" toreign country 2,000,000 210,000
"B foreign country 2,500,000 30,000
155. The allowabie tax credit is
153. One of the foliowing is not a remedy against double taxation
A. P 240 000 C P 261,250
A. esiate tax cedit B. 235,000 D. 266,250
Bvanishing deduction
C delivery of property from fiduciary heir to fideicommissary in a fideicom
missary substitution
Dtransfer for pub!ic purpose B
See the solution in Number 155 below.

D 156. The estate tax due after tax credit is


A P235,360 C. P210,000
154.
Bong0, single, died in the Philippines leaving a net estate in the Philippine of B
380,000 D 153,750
P1,200,000 and P1,800,000 in the United States. His estate in the un
States paid an estate tax of P25,000 in that country. The Philippine estate
payable is
C
136 CPAREVIEWERIN TAXATION=Ampongan ESTATE TAXATION=Multiple Choice Questions 137
Net estate, Philippines P1,500,000 B
Net estate, "A" foreign country P2,000,000
Net estate, "B" foreign country 2.500.000 4.500,o00
Which of the rolIowing statements is correct?
Taxabie estate 6,000,00o 59.
Rate of tax Statement 1: In case the availa cash of the estate is not
6% sufficient to pay
Estate taX 360,000 He total tax liability, the estate may be allowed to
pay the tax by instailment.
Tax credit
Tax paid, "A" P 21000Q Statement 2: In case the estate tax
has been paid by installment, it should not
exceed two (2) years from the date of death.
Limit (2/6 x 360,000) 120.000
Credit allowed (lower) 120,000 A Statement 1 oniy C. Statement 2
Tax paid, "B only
30.000 B Statements 1 and 2 D. Neither 1 nor 2.
Limit (25/6 x 360,000) 150.00
Credit allowed (lower) 30.000
Total credits, 1 limitation 150.000 B
2ndCredit
limit, aliowed(lower
(45/6 x360,000) 270.000
150.000
Estate tax due after tax credit 10,000 160. An estatetax retum is not necessary in the folowing instance

157. What period (from the time of death) is given to file the estate tax
retumn?
A. Transfer of motor vehicle valued at P80,000
B. The transfer is exempt from tax, the gross value of the stocks is
How about the maximum period of extension for filing the return? P300,000
C. The net taxable estate is worth P250,000
Filing Extension D. Donation of cash worth P5,000
6 months 30 days
B. 6 months 60 days
D
year 30 days
D. year 5 years
161. First Statement: The estate tax retum should be filed with the authorized

C 8gent bank, Revenue District Officer Collection Agent or duly authorized


treasurer of the municipality in which the decedent was domiciled at the time
of his death
158. Which of the foitowing is not corect?
Second Statement: If the decedent was a nonresident, not a citizen of the
the estate tax return may be filed with the Commissioner of Intermai
A. A statement duly certified by a CPA is necessary if the estate tax retur iippines
Revenue.
shows a gross value exceeding P5,000,000
B. Extension of time for filing maybe A. Both statements
granted by the Commissioner of
ine are true.
Revenue but it must not exceed 3 months. BBoth statements are false.
C. Shares not traded in the stock exchange shall be valued at its book value
First statement is true while second statementis false,
on the valuation date. First statement isfalse while second statement is true.
tax retun is necessary in case the transfer is to estate
D.Aestate
tax
subject
A
138 CPA REVIEWER IN TAXATION-AmDono ESTATE TAXATION- Multiple Choice Questions
ngan 139
P91,800
162. Case 1- Carl filed an estate tax
return on the date prescribed by law hiepaid A. 97,400
C.P133,110
the tax due of P40,000 after said date. Carl is subject to a surchara B 141,230
P10,000. of

Case 2- If Cari filed the retum and paid the tax of P40,000 with
an A
revenue district officer other than those with
wnom the retum is required ternal
filed after the prescribed date, Carl is subject to the totaB to be Gross estate
surcharge of P30.cm Less: Deductions P10,000,000
A Penalty in Case 1 is correct, penalty in Case 2 is wrong. Ordinary-
B. Penalty is Case 1 is wrong. penalty in Case 2 is corect.
C. Penalties imp0sed on both cases are correct. Claims 2gainst the estate P 100,000
D. Penaities imposed on both cases are wrong. Transfer for public use 200.000
Losses 300,000
Unpaid taxes 20,000
A Unpaid mortgages 50,000 670,000)
Special -

A surcharge of 25% of the basic tax shall be imposed on the Fainily home
(2,500,00o)
following: RA 4917.
a. Late filing of return and late
pay1nent of tax; and Standard 300,000)
b. Filing the return with an unauthorized
person
5.000.0o0)
Net estate 1,530.000
Thus, Carl shall be subject to the following penalties: x Rate of
tax 6%
Case 1-Late filing (P40,000 x25%) Estate tax due
P 10,000 91,800
Case 2- Late filing (P40,00o x 25%)
10,000 164. How much is the estate tax
Filing with unauthorized person (40,000 x 25) 10,000
payable, including increments, if the tax is paid
Total penalties January 10, 2020.
20,000
. P91,800 P133 110
Questions 163 and 164 are based the B 97,400 125,766
on following data.
Decedent, widower, died January 10, 2018 had the foliowing data
Gross estate P 10,000 000
D
Funeral expenses
Judicial expenses 250,000 Tax due
P91,800
Claims against the esiate
150.000 Surcharge (91,80O x 25%) 22.950
Transfer for public use 100,000 Interest (91.800 x 12°% x 1 year) 11,016
Losses 200,000 Payable as of Jan i0, 2020
300,000 125.766
Hospitalization expenses 85,000
Unpaid taxes uestions 165 and 166 are based on the foilowing information
Unpaid mortgages 20,000
Family home 50,000
RA 4917 2,500,000 Vma
was
laion died onJanuary 3, 2018. The estate tax due and payable of P25,000
paid January 3, 2021
300,000 Th e estate tax
163 The estate tax due due, including increments but excluding compromise penalty is
is
A. P 37,250
B. P 41.250
45,000 38,750
140 CPA REVEWER IN TAXATION-AmDor ESTATE AXATION-Multiple Choice Questions
141
A Fven if the estate
tax nave not been paid yet, the BIR Commissioner
C
athorize the bank to aliow Withdrawal of an amount not may
Estate tax due P 25,00o P20,000 by the heirs of A. exceeding
Surcharge (25,000x25%)
Interest (25.000x 12%x2 years)
6,250 nThe manager of BPI must aliow withdrawal upon submission of
6.000 documents and withholding of 6% final tax. pertinent
Estate tax payable 37,250
166. The tax due if the tax is paid on February 3, 2019 (disregard centavos) C
P 25,000 C. P31,505
31,675 31,500 AcO Which of the following statements is FALSE? The estate of the decedent
maybe allowed to pay on instaliment

C A but filing of the return should be made within one year from decedent's
death.
Estate tax due B. within 2 years from decedents deatn.
25,000
Surcharge (25.000 x 25%) but subject io surcharge and interest if payment is made after one year
6,250
Interest (25,000 x 12% x 31/365) 255 from decedent's death
Estate tax payable D. but the frequency, deadline and amount of each instalment shail be
31,505
indicated in the estate tax retun
167. Which statement is WRONG? If the executor includes a bank deposit in the
gross estate and the estate tax thereon paid
C
the estate tax on the bank deposit shali be refunded and the withdrawal
shall be subject to 6% final withholding tax. 170.
B the executor shall present the eCAR issued for the estate
prior to
incase of insufficiency cash
of for immediate
the
due, the following options are allowed, except
payment of the estate tax

withdrawing from the bank deposit account.


Such withdrawal shall no longer be subject to instaliment within one year from the date of decedent's death
.
the withholding tax.
the final tax withheld on the withdrawai of the deposit shail not be
A.BCash
Cash installment within two years from the date of decedent's death
refunded or credited on the tax due of the net taxable estate C.Cash instaliment within two years from the date of decedent's death but
Subject to surcharge and interest on payments made after one year
DPartial dispositio!n of estate and application of its proceeds to the estate
A tax due

168. On September 21, 2019, the manager of BP! upon reading the
obituaiy L C
announcing
A's
the death of Mr. A, refused to allow the heirs of the decedent
Withdraw deposit. A week later, immediately after said denial, the heirS Su
BPI and its manager to
compel them to release the money h ore Commissioner of Internal Revenue may extend the time for the payment
act was arbitrary and a denial of their property's constitutionalalleging
the following statements is rights wniG
h of estate tax in case the estate is settled
NOT CORRECT?
A. The bank should Judicially Extrajudicially JudiciallyExtrajudicially
allow withdrawal from A's deposit account pon
compliance of required documents. u A.. 2 years 5 years C. 5 years 2 years
B.The case brought B. 3 years D 4 years 2 years
by the heirs year
against the
bank wil in
prosper cou
142 CPA REVIEWER IN TAXATION-AmDon ESTATE TAXATION-Multiple Choice Questions
.143
D
C

notice of death and a CPA certificate, respectively, maybe


172. Masaru died on February 24, 2019. The executor of the estate filed n . 175. A
value of the
estate exceeds -
required if the
tax retum on June 23, 2020. Within the period for payment, the execut
requested for an extension of time for paying the tax which was Notice Certificate
approve by Notice
the CIR. Assuming that thetax payabie in the return is P20,000, how
A. P 20,000
P 2,000,000 C. P 200,000 P Certificate
should be paid by the estate? much ,20,000 200,000
2,000,000
D. Not required 5,000,000
b.
A. P 25,782 C. P 32,000
B. 28,000 29,000
D

A 176 Which of the following statements is false? f the decedent died in 2020
Estate tax due per return P 20,000o filing of the estate tax return must be made within one year from
A.
Interest (20,000 x 12% x 119/365) 782 decedent's deeth.
Estate tax due and payabie 20,782 3. notice of death is required
Any amournt paid after the statutory date, but within the extension C. estate tax may be paid on instaiment but within 2 years from decedent's
period, shail be subject to interest but not to surcharge. death.
Da CPA certificate is required if the gross vaiue ofthe estate is more than
P5,000,000.
173. Which of the following is not true?
A The estate tax shouid be paid before deiivery of the distributive sihare in
the inheritance to any heir or beneficiary. B
When there are two executors, both of them are severaliy liable for the
payment of the estate tax.
177. Which of the following statements is not required to accompany the estate tax
C
D
The heir or beneficiary has the primary obligation to pay the estate tax
The liabitity of the heir in the payment of the tax shall in no case exceed
return?

the value of his share in the inheritance. A. temized assets with corresponding value
B. Itemized deductions from gross estate
C. Estate tax due and payable
D. temized income and expenses of the decedent.

The executor or administrator ofan estate has the primary obligationto pay
the estate tax but the heir or beneficiary has subsidiary liability for the D
payment of that portion of the estate which his distrib1ntive share bears to
the value of the total net estate.

174. The estate tax retum should be


accompanied bya certificate of an indepernae
CPA if the gross estate of a decedent is:

P5,000,00o C. Over P2,000,000


2,000,000 or over D Over P5,000,000
144 CPA REVIEWER INTAXATION- ESTATE TAXATION=Mutiple Choice Questions 145
Probiem B
LUCIA BANDANA Y FANDANO, a resident of No. 25 A. Lake St, Sar
died on June 12, 2019. Information pertaining to the decedent foliow n Cty 30B is
2. ltem
Taxpayer Information: P5,300,000 C P9,300,000
AB. 4,000,000 D. 8,300,000
TIN of Lucia Bandana y Fandano 569-628-963
TIN for ONETT
Tax paid in previously filed retum
659-875-340
P 20,000

Excusive properties:
A
Toyota Fortuner, P 1,200,000
is
Residential land (family home) (Land area, 1,023 square meters, Zonal 3. Item 34C
value-P4,000,000; Tax Declaration No. 16-0398, FMV P3,215 9001
-

A P7,600,000 C P17,310,000
Agricultural land, Taytay, Rizal (Area, 9,423 square meters; TCT No B. 10.200,000 D 17,800,000
19-23461-569: Tax Declaration No. 16-61325, FMV-P3,780,000
Zonal value P5,000,000)

Community properties: D
House (family home) (Tax Dec. No. 16-0426, FMV, P5,300,000)
Cash, Metrobank account No. 508-9-60805432-9), P 510,0000 4. 1tem 37D is
Interest in Alba, Bandana & Associates, a professional
partnership A. P5,000, 000 C P11,830,000
registered in No. 246 Aurora Boulevard, San Juan City. P2.000,000 B. 11,105,000 D 10,830 000
Deductions:
Judicial expenses P 45,000
Funeral expenses 150,000 C
Notes payables (loan document worth P50,000 is 100,000
notarized)
Claims against insolvent persons (50% is collectible) 5. Item 39 is
120,000
Death benefits under RA 4917 180 000 A. P4,000,000 C P 3,745,000
Medical expenses (1/2 is not
supported by receipts) 550,000 B. 4,055,0000 D 7.490,0000
The entire cash deposit were withdrawn by the surviving spouse during tne
wake of the deceased.
C
The amended return was filed and the
tax paid on July 12, 2020 6. Item 40 is
PLEASE REFER TO BIR FORM
1801 A. P6,320,000 CP2,115,000
1. Item 29A is B. 2,370,000 D 2,720 000
A. P9,000,000
P 8,000,000
B. 5,000,000 D 13,300,000 C
146 CPA REVIEWER INTAXATION-A TAXATION = Multipe ChoiceQuestions 147
7 tem 18 is
Ampongan STATE
ES
Solution:
P 140,700 P2,345,000
Exclusive Coimunal Total
126,900 142,200 Agricultural land P5.000,000
Residential land 4,000,000
Farnily house P5,300,000
B Toyota Fortuner i,200,000
Interest in partnership 2,000,0000
an insolvent 120,000
8 tem 19C Claim against -J80.00.
under RA 4917
Death benefits
P 25,000 P 20,0000 Gross estate 10,200,000 7.600.00 P17,800,000
140,700 None Deductions
Ordinary-
Note payabie 50,000
60,000 10,000
C Bad debts (50%)
17,690,000
Estate after ordinary deductions
Special -
9 tem 22 is (5.000,00o)
Standard deduction
A P 106,9000 P115,700 Family home
House (5.300,000/2)
2,650,000
B. 25,000 117,200 6,650,000)
4.000,000
Land
180.000)
RA 4917 5.860,00o
A Net estate
SSS (7,600,000-110,000)/2
3.745.0000
2.115.000
Net taxabBe estate
10. Item 23A is
Rate 126,900
P 35.175 Estate tax die 20.000
P57,850O previousBy paid
26,725 D 29,300 Tax credit -

tax 106.900
Tax payable
Add: Penalties 26,725.0bo
Surcharge (106,900 x 25%) 27.776
B interest (106,900 x 12% x 30/366)
1051.48
4.676
Total amount payable
11. tem 23B is
A. P 1,921.31
B
1,054.36 P1,172.00
1,051.48

You might also like